Предисловие
I олимпиада
II олимпиада
III олимпиада
IV олимпиада
V олимпиада
VI олимпиада
VII олимпиада
VIII олимпиада
IX олимпиада
X олимпиада
XI олимпиада
XII олимпиада
XIII олимпиада
XIV олимпиада
Ответы, указания, решения
Содержание
Text
                    ВЫПУСК
fk Щ
Библиотечка КВАНТ
Интеллектуальный марафон*
у\АТ
-г1 г-1
'*. _ тЛ
'Г
Ш Р О


БИБЛИОТЕЧКА КВАНТ ВЫПУСК Приложение к журналу «КВАНТ»№5/2006 ОЛИМПИАДЫ «Интеллектуальный марафон» А. А Егоров, Ж.М.Раббот МОСКВА 2006
УДК 373.167.1:51+51(075.3) Серия ББК 22. 1я721 «Библиотечка «Квант» основана в 1980 г. РЕДАКЦИОННАЯ КОЛЛЕГИЯ! Б.М.Болотовский, А.А.Варламов, В.Л.Гинзбург, Г.С.Голицын, Ю.В.Гуляев, М.И.Каганов, С.С.Кротов, С.П.Новиков, Ю.А.Осипьян (председатель), В.В.Произволов, Н.Х.Розов, А.Л.Стасенко, В.Г.Сурдин, В.М.Тихомиров, А.Р.Хохлов, А.И.Черноуцан (ученый секретарь) ЕЗО Егоров А.А., Раббот Ж.М. Олимпиады «Интеллектуальный марафон». Математика. -М.: Бюро Квантум, 2006. — 128с. (Библиотечка«Квант». Вып. 97. Приложение к журналу «Квант» № 5/2006.) ISBN 5-85843-062-7 Книга представляет собой сборник математических задач, а также вопросов по истории математики, предлагавшихся на Международных олимпиадах «Интеллектуальный марафон» на протяжении пятнадцати лет. К большинству задач даются подробные решения или краткие ответы. Для старшеклассников средних школ, лицеев и гимназий, для членов и руководителей математических кружков, а также для всех любителей решать интересные задачи. |ЁрР| ББК22.1я721 ISBN 5-85843-062-7 © БюР° Квантум, 2006
ПРЕДИСЛОВИЕ Эта книга содержит задачи по математике и вопросы из истории математики, предлагавшиеся участникам Международной олимпиады «Интеллектуальный марафон» в разные годы. Большинство задач заимствованы (после частичной переделки) из различных источников, а остальные придуманы специально для олимпиады. Все задачи снабжены ответами, решениями или указаниями. Международная олимпиада «Интеллектуальный марафон» проводится Международным интеллект-клубом «Глюон» с 1991 года. О том, как она организована, подробно рассказывается в книге «Олимпиады «Интеллектуальный марафон». Физика» (Библиотечка «Квант», вып. 96). Остановимся лишь на некоторых аспектах проведения олимпиады. С 1991 года проводятся письменный индивидуальный и устный командный туры по математике; с 1994 года добавился устный командный тур «История научных идей и открытий». Письменный тур - это обычная олимпиада, в ходе которой каждый участник решает 7 задач (в первые годы количество задач было больше). При проверке работ каждой задаче присваивается количество баллов, примерно обратно пропорциональное количеству ее решивших, так, чтобы сумма баллов по всем задачам была равна 100. Результат каждого участника выражается целым числом баллов. После тура проводится разбор задач с анализом ошибок и обсуждение работ с участниками. Устный тур - это командное соревнование. Команды сидят за отдельными столами. Член жюри у доски формулирует задачу, разъясняет ее условие и сообщает, сколько баллов она стоит и сколько времени (от 3 до 15 минут) отпускается на ее решение. И то и другое зависит от сложности задачи. Общее количество задач в разные годы колеблется, а сумма баллов по всем задачам устного тура всегда равна 100. По истечении отпущенного времени команды сдают листки с решениями, жюри разбирает у доски решение этой задачи, комментирует результаты проверки предыдущей задачи и сообщает количество баллов, полученных за нее командами. Для
этого на доске рисуется таблица, в которую эти баллы записываются. Затем формулируется следующая задача и так далее. Точно так же проводится устный командный тур по истории научных идей и открытий. В этом соревновании вопросы по математике и физике чередуются, а их общее количество равно 10. Необходимо отметить своеобразие вопросов этого тура. Поскольку школьники практически не знают историю математики, часть вопросов выглядит так. Сначала очень кратко рассказывается какой-нибудь эпизод из истории математических открытий, а затем предлагается решить задачу, связанную с этим эпизодом, т.е. «поучаствовать» в сделанном некогда открытии (см., например, задачи 237, 258, 259 и др). Очень важной частью олимпиады «Интеллектуальный марафон» является общение участников друг с другом и с членами жюри - известными физиками, математиками и педагогами - в «кулуарах» олимпиады. Школьники узнают много нового, расширяют свой кругозор, открывают для себя новые горизонты. Ведь практически все задачи, предлагающиеся на олимпиаде, нестандартны, в каждой есть своя «изюминка». Мы надеемся, что эта книга вызовет интерес у школьников, увлекающихся математикой, у учителей, которые смогут использовать ее в своей работе, и вообще у всех любителей математики. О замеченных опечатках, неточностях и иных недостатках книги просим сообщать по адресу редакции журнала «Квант».
I ОЛИМПИАДА Ашхабад, 1991 Письменный тур 1. Докажите, что число 53-83-109 + 40-96-96 является составным. 2. Найдите сумму: 1 1 1 ' 1 з+ 4+ ♦ п 3. В треугольнике ABC расстояние от центра описанной окружности до стороны АВ равно d, а /ЛВС = 60° . Точка D на стороне ВС такова, что BD = АВ/2. Найдите длину отрезка CD. 4. Решите уравнение (Ах3 - 8) (3sin* -1) + (2*3 - 4)sin x = 0. 5. Решите в целых числах уравнение \9х3 -9\у2 = 1991. 6. Докажите, что если положительные числа a, b, k, n удовлетворяют неравенству ab > ka + nb, то а + Ъ > (yfk 7. На катете АС прямоугольного треугольника ABC отложен отрезок AD = В С, а на катете ВС - отрезок BE = CD. Найдите угол между прямыми BD и АЕ. 8. Рассматриваются всевозможные параболы у = х2 + рх + + q , пересекающиеся с осями координат в трех различных точках. Докажите, что окружности, описанные около треугольников с вершинами в этих точках, имеют общую точку.
9. Каждая грань кубика разделена на 9 квадратов и в каждый квадрат вписано некоторое число. Известно, что сумма пяти чисел, вписанных в любой квадрат и в четыре соседних с ним, равна 17. Могут ли все вписанные числа быть целыми? 10. Числа а, Ь, с - корни уравнения х3 - Зх +1 = 0 , причем а <Ъ < с. Докажите, что Ь2-а = с2-Ь = а2-с = 2. 11. На сторонах АВ, ВС и АС данного остроугольного треугольника ABC во внешнюю сторону построены как на основаниях попарно подобные равнобедренные треугольники А'ВС , АВ'С , ABC . Докажите, что прямые АА', ВВ' и СС пересекаются в одной точке. 12. Докажите, что из любых четырех положительных чисел можно выбрать два - х и у - так, чтобы выполнялись неравенства 2-Тз. 0* \ 1 + х + у + 2ху Устный тур 13. Какое из двух чисел больше: VT990+V1992 или 2>/T991 ? 14. Медианы AAV ВВХ и ССХ треугольника ABC пересекаются в точке М. Радиусы окружностей, вписанных в треугольники АМСХ и ВМСХ , равны. Найдите площадь треугольника ABC, если АС = 5, АВ = 8. 15. Пусть Sk - сумма первых k членов арифметической прогрессии. Найдите Sp+q , если Sp = q , Sq = р (q * р). 16. На плоскости дан отрезок АВ. Найдите геометрическое место точек С таких, что в треугольнике ABC высота ВН равна медиане AM. 17. Числа а, Ъ и с удовлетворяют системе а + Ъ + с = 1, а3 + Ь3 + с3 = 1. Докажите, что одно из этих чисел равно 1. 18. Какое наибольшее количество точек пересечения могут иметь контуры двух четырехугольников? 19. При каких п неравенство х\ + х\ +... + х\ > хп (*, + х2 +... + *„_,) справедливо для любых хих2,...,хп? 20. На доске выписаны числа 1, 2, ..., 1991. Разрешается взять несколько чисел, сумма которых s делится на 5, вычерк-
нуть их и вписать число 5/5. Можно ли добиться того, что в результате на доске останется единственное число 1? 21. Найдите наименьшее значение суммы vl + х2 + yl + у2 + + VI + 22 , если числа х, у, z неотрицательны их + у + г=\. 22. К окружности со из внешней точки Р проведены касательные PS и РТ (S и Т - точки касания) и внутри со построена дуга ST окружности с центром Р. Через точку А этой дуги проведены прямые AS и AT, пересекающие окружность со в точках В и С. Докажите, что: а) ВС - диаметр окружности со; 6) прямые ВС и АР перпендикулярны.
II ОЛИМПИАДА Зеленоградск, 1992 Письменный тур 23. Какое из чисел больше: 19931991 • 19911993 или 19923984 ? 24. Докажите, что 13 _J_ _J_ _J_ И 12 < 1993 + 1994+"+7968 < 6 * 25. Решите уравнение (х2 - х -1)3 + (2х2 - х - 7)3 = (Зх2 -2х- 8)3. 26. В треугольнике ABC проведены медианы ВМ и CN, пересекающиеся в точке О. Найдите АС, если известно, что в четырехугольник AM ON можно вписать окружность и ВС = 2, NC= у/3. 27. а) Докажите, что сумма квадратов любых 10 последовательных натуральных чисел не является полным квадратом. б) Найдите 11 последовательных натуральных чисел, сумма квадратов которых является полным квадратом. 28. Пусть а - корень уравнения х2 + рх + q = Q (q фО), ар- корень уравнения х2 - рх - q = 0 . Докажите, что между числами аир найдется корень уравнения х2 - 2рх - 2q = 0 . 29. В треугольнике ABC через середину К биссектрисы BD проведена прямая, перпендикулярная BD и пересекающая прямую АС в точке Е. Докажите, что DE2 = АЕ • ЕС . 30. В каждой клетке таблицы m x n стоит единица. Разрешается взять произвольный квадратик 2x2 и поменять в нем знаки у всех чисел. При каких тип можно с помощью таких операций получить «шахматную» расстановку знаков в таблице? Устный тур 31. Некто взял с собой в командировку кусок мыла. Вечером 7-го дня командировки он обнаружил, что размеры куска сократились в 2 раза. Хватит ли ему мыла на
оставшиеся 2 дня? (Предполагается, что каждый день он использует одно и то же количество мыла.) 32. Каким наименьшим числом разрезов можно разделить куб размерами 3x3x3 на единичные кубики? 33. Могут ли высоты треугольника быть меньше 1 мм, а площадь больше 1 км2 ? 34. Последовательные стороны четырехугольника равны 9, 1, 8, 12. Найдите угол между его диагоналями. 35. Какое наибольшее количество: а) королей; б) коней можно поставить на шахматную доску так, чтобы они не били друг друга? 36. Существует ли прямоугольный треугольник, две медианы которого перпендикулярны? 37. Десятичная запись числа 2" оканчивается тремя одинаковыми цифрами. Какими? оо тт , sin Г sin3° ^ 38. Что больше: или sin 2° sin 4° 39. Найдите сумму первых десяти чисел, общих для двух арифметических прогрессий: 1, 6, 11, ... и 3, 10, 17, ... 40. Существуют ли 2 пятиугольника с общими вершинами и такие, что ни одна из сторон одного из них не является стороной другого?
Ill ОЛИМПИАДА Старый Петергоф, 1993 Письменный тур 41. Найдите все четырехзначные числа, являющиеся полными квадратами и такие, что их первые две цифры совпадают и две последние тоже совпадают. 42. Можно ли расставить числа 1, 2, ..., 81 в клетки квадратной таблицы 9x9 так, чтобы суммы чисел во всех горизонтальных рядах были одинаковыми? 43. На сторонах ВС и CD квадрата ABCD со стороной 1 взяты точки М и N такие, что периметр треугольника CMN равен 2. Найдите угол MAN. 44. Решите уравнение 45. В треугольнике ABD угол ABD равен 120°, на стороне AD взята точка С такая, что АВ = CD = 1 и угол ABC равен 90°. Найдите АС. 46. Центральный банк выпустил в обращение монеты достоинством в 15, 20 и 48 рублей и изъял из обращения все другие деньги. а) Докажите, что любое целое число рублей можно уплатить этими монетами, быть может, получив сдачу. б) Докажите, что любую сумму, начиная с некоторого N, можно уплатить и без сдачи, и найдите наименьшее возможное N. 47. Докажите, что число 1280000401 является составным. 48. Пусть ava2,...,an - попарно различные натуральные числа. Докажите неравенство > ах +а2 +... + <!„ 1 + 2 + ... + 71 Устный тур 49. Найдите сумму 1+2-3-4 + 5 + 6-7-8+... - 1992 + 1993. 10
50. В трапеции с основаниями 3 и 6 проведен отрезок длины 5, параллельный основаниям и делящий ее на две трапеции. Найдите отношение площадей этих трапеций. 51. Найдите сумму квадратов корней уравнения [х2 + 2xf -1993 [х2 + 2х) +1995 = 0 . 52. Может ли квадрат целого числа оканчиваться цифрами ...341? 53. Может ли сумма 1993-х последовательных нечетных чисел быть 1993-й степенью некоторого числа? 54. Площадь выпуклого четырехугольника ABCD равна 1/2. Кроме того, АВ + BD + DC < 2 . Найдите АС. 55. Какое из чисел больше: 3100+4100 или 5100? 56. Найдите наименьшее значение выражения и выясните, при каких (х; у) оно достигается. 57. Можно ли разрезать квадрат на 3 подобных между собой и попарно неравных прямоугольника? 58. Решите уравнение х4 + 8х - 7 = 0 .
IV ОЛИМПИАДА Ростов-на-Дону, 1994 Письменный тур 59. Найдите трехзначное число, куб которого оканчивается на три семерки. 60. Точка внутри выпуклого четырехугольника с площадью S соединена с его вершинами. Найдите площадь четырехугольника, вершины которого являются точками пересечения медиан образовавшихся четырех треугольников. 61. Решите систему уравнений 62. Можно ли, пользуясь только операциями сложения, вычитания и умножения, составить из многочленов f(x) и д(х) выражение, равное х, если: а) f(x) = x2+x, д{х) = х2+2', б) f(x) = 2x2+x, д{х) = 2х; в) f(x) = x2+x, д(х) = х2-2? 63. Окружности с центрами О и О' расположены на плоскости одна вне другой. Касательная, проведенная из точки О ко второй окружности, пересекает первую окружность в точках Л и Б, а касательная из О' к первой окружности пересекает вторую окружность в точках А' и В' , причем точки А и А' лежат по одну сторону от прямой ОО'. Зная расстояния АА' = а и ВВ' = Ь , найдите ОО'. 64. В правильном: а) пятиугольнике; б) шестиугольнике проведены все диагонали. Около всех вершин и всех точек пересечения диагоналей написаны единицы. Разрешается поменять знаки всех чисел, стоящих на любой линии (стороне многоугольника или диагонали). Можно ли за несколько таких операций поменять знаки всех чисел? 12
65. Строго возрастающая функция f(k), заданная на множестве натуральных чисел (k = 1, 2, 3, ...) и принимающая натуральные значения при всех k > 1, удовлетворяет условию f(f(k)) = 3k. Найдите /"(1994). Устный тур 66. Яйцо варится 4 минуты. Как сварить его, имея двое песочных часов, рассчитанных на 3 минуты и на 5 минут? 67. На прямой / отмечена точка Р. С помощью циркуля и линейки восставьте перпендикуляр к прямой / в этой точке, проведя при этом наименьшее количество линий. 68. Коммерческий директор фирмы «Не обманешь - не продашь» на собрании акционеров заявил, что в течение последнего года за любые пять последовательных месяцев фирма терпела убытки, но в целом за год получила доход, и большой. Могут ли акционеры привлечь директора к суду за обман? 69. Неравенство х2 + рх + q > О (р и q - целые числа) выполняется при всех целых х. Докажите, что оно верно и при всех действительных х. 70. Разгадайте числовой ребус КРУГ2 =КР***УГ (разными буквами обозначаются разные цифры). 71. Найдите площадь треугольника АВСУ если его медиана AM и биссектриса BL перпендикулярны, причем AM = a, BL = Ь. 72. Рассматриваются всевозможные четырехугольники с данными диагоналями / и m и углом а между ними. Какой из них имеет наименьший периметр? 73. Положительные числа х, у и z удовлетворяют неравен- 111^ ствам xyz >1, х + у + z < —\- —\-— , Сколько среди них чисел, х у z меньших единицы? 74. Заполните пропуски так, чтобы стало истинным высказывание: «В этом предложении цифра 0 встречается ... раз, цифра 1 встречается ... раз, цифра 2 - ... раз, цифра 3 - ... раз, ..., цифра 9 - ... раз». 75. Существует ли число, сумма цифр в десятичной записи квадрата которого равна: а) 1994; б) 1993. 13
История научных идей и открытий 76. Приведите примеры математических теорий или понятий, возникших в рамках «чистой математики» и нашедших спустя годы прямое применение в естественных науках и практике. 77. 26-го ноября математики отмечали день памяти одного своего выдающегося коллеги. В мемуаре «Теория механизмов, известных под именем параллелограммов» он получил важнейшие результаты по математической проблеме, которую он там же и поставил и к разработке которой многократно возвращался. О каком разделе математики и о какой задаче идет речь? Что изучает эта наука, что означает и от кого пошло ее название? Какая работа этого ученого стала основополагающей в новой области знания? 78. 26 ноября - день рождения крупного математика, известного, в частности, построением модели броуновского движения, названной его именем, но более всего тем, что он стал, можно сказать, отцом новой науки. О ком и о чем здесь идет речь? Что изучает эта наука, что означает и от кого пошло ее название? Какая работа этого ученого является основополагающей в новой области знаний? 79. При каких обстоятельствах математики испытали действительную нужду в мнимых числах? Кто были эти математики? 80. Имя, точнее прозвище, этого математика превратилось в общеизвестный математический термин, а из названия одного из его трудов возникло название крупного раздела математики. О ком и о чем идет речь? Назовите полное имя этого математика. 81. На надгробии того, кого называли «Rex Mathematicorum», изображен правильный N-угольник. Почему? Кто покоится в этой могиле? Чему равно N1
V ОЛИМПИАДА Варна (Болгария), 1995 Письменный тур 82. Найдите сумму цифр числа 99...9 3 . п девяток 83. Найдите углы треугольника, подобного треугольнику, образованному основаниями его высот. Рассмотрите все возможные случаи. 84. Числа 1, 2, 3, ..., 50 каким-то образом разбили на 10 пятерок и в каждой пятерке взяли среднее по величине число. Каковы: а) наибольшая; б) наименьшая суммы этих чисел? 85. Решите систему уравнений л -г j j — о, х +У у- ——т = 0- х2+у2 86. Сколько существует квадратных уравнений х2 - рх - q = 0 (р и q - натуральные), имеющих положительный корень, который не превышает: а) 5; б) любое натуральное п? 87. Вершины параллелограмма Ку L, М, N площадью 30 лежат соответственно на сторонах АВ, ВС, CD, DA четырехугольника ABCD и делят его стороны в одном и том же отношении 1/3, считая по часовой стрелке. Известно, что около четырехугольника ABCD можно описать окружность. Найдите ее радиус, если АВ = 6. 88. При каких тип прямоугольный лист клетчатой бумаги из т х п клеток можно разрезать на фигурки, состоящие из 4 клеток и имеющие форму буквы Г? Устный тур 89. Можно ли нарисовать на плоскости 1000 окружностей так, чтобы любые две окружности касались друг друга? Л 1 1 1 90. На доске записаны дроби ^^'"^'Т'-'Тп • а) Можно ли 15
расставить между ними знаки + и - так, чтобы полученная сумма оказалась равна 0? 6) Какое наименьшее количество чисел нужно вычеркнуть, чтобы после некоторой расстановки знаков получилась сумма, равная О? 91. Хитроумный часовщик приделал к оси правильно идущих часов третью стрелку, которая в каждый момент времени делит пополам угол между часовой и минутной стрелками. Сколько оборотов сделает третья стрелка за сутки (в полночь все три стрелки совпадают)? 92. Какое из двух чисел больше -л или -л ? 2 + Ц 2 + - 1995 1995 93. На какое наименьшее количество выпуклых четырехугольников можно разрезать выпуклый 17-угольник? 94. В арифметической прогрессии сумма Sm первых т членов равна сумме Sn первых п членов (т Ф п). Чему равна sm+n ? 95. Решите уравнение 2х4 + 2уА = 4ху-\. 96. При каких а числа а + >/Г5 и — >/15 целые? а 97. В равнобедренном треугольнике ЛВС (АВ = ВС) проведена биссектриса CD. Перпендикуляр к CD в точке D пересекает прямую АС в точке Е. Найдите СЕ, если AD = а. 98. На некоем острове живут 13 серых, 15 бурых и 17 малиновых хамелеонов. Если встречаются два хамелеона разного цвета, то они одновременно меняют свой цвет на третий. Может ли случиться так, что через некоторое время все хамелеоны будут одного цвета? История идей и открытий 99. Математики древности доказывали формулу (а + b) =a +2ab + b с помощью картинки (см. рис.1). Формулы а2 - Ь2 = (а - b) (a + b), (a- bf = а2 - 2ab + Ь2 они также доказывали с помощью картинок. Нарисуйте их. 16
b a a b Рис. 1 100. Вначале это обозначали так: R2a , R3a, затем так: Va, VVa, потом 2 3 так: V а , V а . Как это обозначается сейчас и кто автор современного обозначения? 101. Известные математики Х1Х-го века Гаусс, Мёбиус, Плюккер, Бессель имели много учеников, однако не учили их математике. Чему и почему они их учили? 102. В 1299-м году власти Флоренции запретили «писать числа столбцами и употреблять индийские цифры». Почему? 103. К математику К однажды пришли братья N. Все они были в шляпах. Когда гости уходили, к величайшему конфузу хозяина одному из гостей шляпы не хватило. Когда братья пришли в следующий раз в шляпах, то после их ухода хозяин обнаружил на вешалке ровно столько шляп, сколько было братьев. В третий раз братья пришли без шляп. Уходя, каждый из них надел одну из шляп, оставшихся после предыдущего визита, но на вешалке осталось столько же шляп, сколько было братьев. Как такое могло произойти. Кто из математиков впервые проник в секреты ситуации, описанной в этой задаче- шутке?
VI ОЛИМПИАДА Созополь (Болгария), 1996 Письменный тур 104. В десятичной записи числа 1/14 вычеркнули 1996-ю цифру после запятой. Что больше: полученное число или 1/14? 105. Из вершины А треугольника ABC опущены перпендикуляры АР и AQ на биссектрисы углов В и С треугольника (или их продолжения). Найдите PQ, если ВС = а, АС = Ь, АВ = с. 106. Найдите два двузначных числа, если известно, что сумма остальных двузначных чисел в 50 раз больше одного из этих двух чисел. 107. В некоторой компании п мальчиков и п + 1 девочка. Может ли случиться так, что все девочки знакомы с различным количеством мальчиков, а все мальчики - с одинаковым количеством девочек, если: а) п = 5; б) п - произвольное натуральное число (укажите все возможные значения я)? 108. На сторонах ВС и CD прямоугольника ABCD взяты точки Е и F соответственно так, что треугольник AEF правильный. Найдите площадь треугольника CEFf если SABE = Sx, a 109. Числа соотношениям х, У и х - Z 1 f — У попарно 1 = */ + - различны 1 = z + — X ' и удовлетворяют Чему может равняться xyzl НО. При каких натуральных п существуют выпуклые п- угольники, которые можно разрезать на конечное число квадратов и правильных треугольников? Устный тур 111. Вычислите V1993-1995 1997 1999+ 16. 112. Найдите площадь трапеции, если известно расстояние d от середины одной из боковых сторон трапеции до прямой, 18
содержащей другую боковую сторону, длина которой равна а. 113. В однокруговом хоккейном турнире участвуют 8 команд, 4 из которых выходят в финал. Какое наименьшее количество очков должна набрать команда, чтобы обеспечить себе выход в финал? (Победа - 2 очка, ничья - 1 очко, поражение - 0 очков). 114. Разрежьте на: а) три; б) пять равных частей квадрат 4 х 4 с вырезанным из него угловым квадратом 1x1. 115. Найдите: а) наибольшее; 6) наименьшее число, делящееся на 11, десятичная запись которого состоит из 10 попарно различных цифр (0 не может быть первой цифрой числа). 116. На сторонах АВ и АС треугольника ABC, вне его, построены квадраты АА'В'В и АА"С'С . Найдите А'А" , если медиана AM треугольника ABC равна т. 117. После вечера танцев каждый из его участников (мальчиков и девочек) сообщил количество танцев, в которых он участвовал: 3, 3, 3, 3, 3, 5, 6, 6, 6, 6, 6, 6, 6, 6, 6. Все ли сказали правду? (Мальчики танцевали только с девочками, а девочки только с мальчиками. От танца к танцу пары могли меняться). 118. Разложите на два последовательных натуральных множителя число 11112222. 119. Два игрока поочередно записывают одну из цифр 1,2, 3, 4, 5 до тех пор пока не получится я-значное число. Второй игрок стремится добиться того, чтобы полученное число делилось на 9. Может ли первый игрок ему помешать, если: а) п = = 20; 6) п = 30? 120. Рассматриваются всевозможные пятизначные числа, цифры десятичной записи которых убывают. Каких из них больше: тех, сумма цифр которых равна 30, или тех, сумма цифр которых равна 31? История научных идей и открытий 121. Эту задачу считал решенной Ферма, опроверг его мнение Эйлер, а к необходимости вернуться к проблеме пришел Гаусс. Что это за задача? (Она до сих пор не решена.) 122. Эта область математики была частью астрономии, затем стала самостоятельным разделом математики, после чего превратилась в учебный предмет. О чем идет речь? 123. Математики древней Греции числа вида 1 + 2 + 3 + ... ... + п называли треугольными, числа 1 + 3 + ... + (2п - 1) - 19
квадратными, числа 2 + 4 + 6+... + 2я - прямоугольными, числа 1 + 4 + 7 + ... + (Зп - 2) - пятиугольными. Почему эти числа так называли? 124. Назовите математические предложения, названные в честь известных математиков (пример - теорема Пифагора). 125. Пифагорейцы не любили некоторое натуральное число, поскольку оно было заключено между числами, выражающими площади целочисленных прямоугольников, периметры которых численно равнялись их площадям. Найдите это число.
VII ОЛИМПИАДА Анталия (Турция), 1997 Письменный тур 126. Существует ли натуральное число п такое, что 5я является пятой степенью натурального числа, 6п - шестой степенью, In — седьмой степенью? 127. Найдите углы треугольника ABC, если известно, что его высота CD и биссектриса BE пересекаются в такой точке М, что СМ = 2MD, ВМ = ME. 128. Решите систему уравнений х + у2 = Z3, x2+y3=z\ 129. Последовательность ап удовлетворяет при любом натуральном п соотношению Найдите я1998 , если я19 = 19 , а97 = 97 . 130. Можно ли разрезать правильный треугольник на 5 попарно не подобных равнобедренных треугольников? 131. Выясните, конечно или бесконечно число решений в натуральных числах уравнения х2 + у2 + z2 - ху - xz - yz = 3 . 132. Пусть xv x2,..., хп - действительные числа такие, что О < хг < 1. Найдите наибольшее значение величины х\ + Х2 + • • • + хп ~ Х\Х2 ~ Х2ХЪ - -•'- хп-\хп ~ хпх\ при: а) п = 3; б) п = 4; в) произвольном п. Устный тур 133. Прямоугольник разделен на 4 прямоугольника, площади которых 3, 4, 5 их (рис.2). Найдите х. pUc. 2 21 3 X 4 5
134. Простое или составное число 210+512? 135. В математическом кружке число девочек больше 40%, но меньше 50% от числа всех участников. Какое наименьшее число участников кружка может быть при этих условиях? 136. Высота прямоугольного треугольника делит его на два треугольника с периметрами Р и Q. Найдите периметр треугольника. 137. Можно ли получить нуль из чисел 12,22,32,...,2002, используя только сложение и вычитание? 138. Квадрат разрезан на 100 квадратов, 99 из них - со стороной 1. Какую площадь может иметь этот квадрат? 139. Найдите наибольшее значение выражения a3b-b3a, если выполнено условие а2 +Ь2 = 1. 140. Можно ли в таблице 6x6 расставить 36 вещественных чисел так, чтобы их сумма была отлична от 0, а сумма чисел в любом прямоугольнике 1x4 была равна 0? 141. Число 1,5 интересно тем, что оно в 4 раза меньше суммы своих цифр. Найдите число, которое в 8 раз меньше суммы своих цифр. 142. Дано 30 гирь с массами 1 г, 2 г, ..., 30 г. Убрали 10 гирь таких, что их общая масса равна — массы всех гирь. Можно ли оставшиеся гири разделить на две кучки одинаковой массы? История научных идей и открытий 143. Кто из математиков получил Нобелевскую премию? 144. Назовите великого математика, который ввел в употребление такие символы, как «л», «е», «sin», «cos» «Z », «/». 145. Какие знаменитые задачи оставили древние математики и когда эти задачи были решены? 146. В XVII-XVIII веках произошли революционные изменения в математике, связанные с созданием математического анализа. В конце XIX века также произошли революционные изменения в математике. С чем они были связаны? 147. Как известно, иногда в названиях встречаются исторические несправедливости. Например, Америку назвали не по имени ее открывателя - Колумба. Назовите известные вам исторические несправедливости в математике.
VIII ОЛИМПИАДА Афины (Греция), 1999 Письменный тур 148. Замените звездочки в записи девятизначного числа 32*35717* так, чтобы оно разделилось на 72. 149. Разрежьте листок клетчатой бумаги размером 8x9 клеток на фигурки, состоящие из четырех клеток и имеющие форму буквы Г (рис.3). 150. Найдите три простых числа, произведение которых в 7 раз больше их суммы. 151. В треугольнике ABC площадью S точка К - середина медианы AM. Прямая ВК пересекает АС в точке L. Найдите площадь треугольника AKL. Рис. 3 152. Решите систему уравнений [у2- 2ху +9 = 0. 153. Можно ли увезти 50 камней, массы которых 370 кг, 372 кг, 374 кг, 376 кг, ..., 468 кг, на семи трехтонных грузовиках? 154. В ромбе ABCD величина угла В равна 40°, Е - середина ВС, F - основание перпендикуляра, опущенного из А на DE. Найдите величину угла DFC. Устный тур 155. Какое из двух чисел: А = 19971998 • 199819" • •19991997 или Я = 19971997- 1998199819991999 больше? 156. Площадь заштрихованного прямоугольника равна Q, площадь прямоугольника ABCD равна Р (рис.4). Чему равна площадь четырехугольника KLMN? К 157. Является ли число 3 + 5-3 +1 простым? 22 \ д \ \ м N Рис. 4 D 23
158. Что вы можете сказать о треугольнике, площадь которого равна -(а2 + Ь2), где а и Ь - две из его сторон? 4v ' 159. Найдите сумму цифр всех трехзначных чисел. 160. Можно ли разрезать выпуклый 17-угольник на 14 треугольников? 161. Два участника шахматного турнира выбыли после 5-го тура, и потому в турнире было сыграно 38 партий. Играли ли выбывшие участники друг с другом? 162. Существует ли натуральное число п такое, что 6п является шестой степенью целого числа, а 8п - восьмой степенью? 163. Какое из чисел: 1 1 1 1 1 Л = + + ... + 1-2 2-3 910 или Рис. 5 164. Сколько прямоугольников имеется на рисунке 5? История научных идей и открытий 165. Каково происхождение терминов «трапеция», «конус», «цилиндр»? 166. Запишите формулой фразу из древнего трактата: «квадрат на отрезке а равен прямоугольнику на отрезках Ь и с». 167. Назовите известные вам наиболее знаменитые числа. Что вы можете сказать о них? 168. Какие знаменитые проблемы древности вам известны? Когда и кем они были решены? 169. Назовите имена известных вам математиков, являвшихся крупными государственными деятелями.
IX ОЛИМПИАДА Протвино, 2000 Письменный тур 170. Найдите сумму цифр всех натуральных чисел: а) от 1 до 2000; б) от 1 до 10". 171. Решите систему неравенств 172. Пусть / - центр окружности, вписанной в треугольник ABC, J - центр окружности, касающейся стороны ВС и продолжений сторон АС и А В (вневписанная окружность), М - середина отрезка //, а /А = а • Найдите ZBMC • 173. Каждый член некоторой компании из п человек поздравил с праздником ровно к человек из той же компании. При каком наименьшем k можно утверждать, что среди данных п человек найдутся два, поздравивших друг друга, если: а) п = 20; 6) п = 21; в) п - любое натуральное число? 174. Можно ли разрезать: а) квадрат; б) равнобедренный прямоугольный треугольник на конечное число равнобедренных трапеций? 175. В треугольнике ABC точка D - середина медианы AM. Прямая CD пересекает сторону АВ в точке N. Найдите CN, если BD = ВМ, a AN = a. 176. Можно ли между числами 13,23,...,я3 расставить знаки «плюс» или «минус» так, чтобы полученная алгебраическая сумма стала равна 0, если: а) п = 1999; 6) п = 2000; в) п = 2001? из таблицы Устный тур 177. Витя и Боря вычеркивают по очереди числа 1 4 7 2 5 8 3 6 9 25
Витя вычеркнул 4 числа, и Боря тоже вычеркнул 4 числа. Оказалось, что сумма чисел, вычеркнутых Борей, в 3 раза меньше суммы чисел, вычеркнутых Витей. Какое число осталось в таблице? 178. Найдите площадь «косого сектора», заштрихованную на рисунке 6, если точки М и Р делят полуокружность AM PC м . р на три равные части, а площадь ^>, полукруга равна S. /Ж \. 179. Наборщик рассыпал неко- I / \ торое число, представляющее ше- А//-^ стую степень натурального числа А* « \q а. Найдите я, если цифры рассы- Рис ^ ® панного числа - 0, 2, 3, 4, 4, 7, 8, 8, 9. 180. Существует ли треугольник, который можно разрезать на три равных треугольника, подобных данному? 181. Существует ли: а) 2002-угольник, описанный около окружности, стороны которого равны (в некотором порядке) 1, 2, 3, ..., 2002; б) описанный 2000-угольник со сторонами 1, 2, 3, ..., 2000 в указанном порядке? 181. Можно ли из последовательности 1, 1/2, 1/3, ... ..., 1/я, ... выделить: а) арифметическую прогрессию с 2000 членов; б) бесконечную арифметическую прогрессию? 183. Расстояние АН от вершины А треугольника ABC до его ортоцентра Н (ортоцентр - точка пересечения высот треугольника) равно радиусу описанной около треугольника ABC окружности. Чему может быть равен угол А? 184. Различные числа х> у, z удовлетворяют равенствам х3 - Зх2 =у3- Ъу2 = z3 - 3z2 . Найдите х + у + z. 185. Внутри сектора с прямым центральным углом расположены два полукруга (рис.7). Найдите заштрихованную площадь, если площадь, ограниченная двумя полуокружностями, равна S. 186. Найдите наибольшее и наименьшее значения выражения (а - vf + (b - uf , если а2 + Ь2 = 1, a v2 + и2 = 4 . 187. Существует ли такая компания, в которой каждый ее участник имеет ровно 6 друзей, а каждые два участника имеют двух Рис. 7 общих друзей? 26
188. Может ли я-значное число (п произведению своих цифр? 1) быть равным ъ Рис, 8 История научных идей и открытий 189. Запишите формулу, которую древние вавилоняне доказывали с помощью картинки, изображенной на рисунке 8. 190. Некий итальянский математик (1180-1240), живший в Пизе, написал несколько книг, по которым потом учились многие поколения математиков. Он , же нашел удивительную последовательность fn: 1, 1, 2, 3, 5, ... а) Как звали этого математика? Как называются открытые им числа? б) Выясните, какие числа fn делятся на три. 191. Французский математик Ро- берваль, пытаясь вычислить площадь под одной аркой циклоиды, нашел некоторую кривую, названную им спутницей циклоиды. Ею оказалась синусоида (в современных обозначениях это график функции у = 1 - cos x ). Найдите площадь, ограниченную этим графиком на промежутке [0; 2л] и осью Ох, не пользуясь интегралом, т.е. так же, как это делал Роберваль. 192. Некоторые математики XVII века полагали, что сумма ряда 1-1 + 1-1 + 1-... равна 1/2. Восстановите их рассуждения и, пользуясь сходными соображениями, найдите «сумму» 1-2 + 22-23+... 193. Формула для площади треугольника S-—ah была известна в глубокой древности и доказывалась разрезанием треугольника на части, из которых можно сложить параллелограмм. При выводе формулы объема пирамиды Архимеду пришлось воспользоваться соображениями, предвосхитившими появление в далеком будущем понятия интеграла. В связи с этим возникла задача: можно ли разрезать тетраэдр на конечное число многогранных частей, из которых удастся сложить параллелепипед? Задачу о возможности такого разрезания поставил в 1900 году один из величайших математиков рубежа XIX- XX веков. Как звали этого математика, как формулировалась поставленная им проблема, кто и когда ее решил?
X ОЛИМПИАДА Протвино, 2001 Письменный тур 194. Можно ли число I2 + 22 +... + 20012 представить в виде суммы: а) 2000; 6) 1999 различных квадратов целых чисел? 195. В треугольнике ABC угол А равен 50°, а угол С равен 70°. На сторонах АВ и ВС взяты, соответственно, точки D и Е такие, что ZACD = ZCAE = 30° . Пусть М - точка пересечения отрезков АЕ и CD. Найдите: а) угол АВМ\ 6) угол CDE. 196. Найдите все простые числа р, для которых являются простыми числа р2 - 2 , 2р2 -1, Зр2 + 4 . 197. Решите систему уравнений \х6 + х2 = 8у3 + 2у. 198. В прямоугольном треугольнике ABC ( Z.C = 90° ) проведена высота CD. Биссектриса угла В пересекает биссектрису угла ACD в точке Е, а биссектриса угла А пересекает биссектрису угла BCD в точке F. Найдите EF} если радиус вписанной в треугольник ABC окружности равен г. 199. Какое наибольшее количество полосок: а) 1x5; 6) 1x6; в) 1x7 можно вырезать по линиям сетки из листа клетчатой бумаги 27 х 34 ? 200. На вечеринку пришли к супружеских пар. При встрече некоторые участники вечеринки обменялись рукопожатиями (естественно, супруги друг другу руки не пожимают). После этого мистер Браун спросил у всех остальных участников о числе сделанных ими рукопожатий. Все названные числа оказались различными. Сколько рукопожатий сделала миссис Браун, если: а) к = 5; 6) k - любое натуральное число? Устный тур 201. Лев съедает овцу за 1 час, медведь - за 2 часа, волк - за 3 часа. За сколько времени съест овцу компания из льва, медведя и волка? 28
202. Может ли трехзначное число иметь 25 делителей? 203. Что больше: а + Ъ или с + hc, где а и Ь - катеты, а с и hc - гипотенуза и высота прямоугольного треугольника? 204. Может ли общая часть (пересечение) треугольника и четырехугольника быть восьмиугольником? 205. Найдите 100 попарно различных натуральных чисел, сумма которых равна 5051. 206. По окружности расставлены числа, как показано на рисунке 9. Разрешается прибавлять или вычитать по 1 к любым: а) двум; б) трем подряд стоящим числам. Можно ли с помощью таких операций добиться того, чтобы все числа стали равными? 207. Всегда ли из бесконечной последовательности различных положительных чисел ах,а2,...,ап,... можно выбрать несколько чисел, являющихся длинами сторон некоторого многоугольника? 208. Числа 1, 2, 3, ..., 100 расставили в таблице 10x10, как показано на рисунке 10. После этого перед некоторыми из них поставили знаки «минус» так, что в каждой строке и в каждом столбце оказалось ровно 5 «минусов». Чему может равняться сумма всех чисел полученной таблицы? 209. Припишите к числу 2001 три цифры справа так, чтобы полученное семизначное число делилось на 7, 9, 11. 210. После окончания волейбольного турнира (каждая команда играет с каждой) оказалось, что для любых двух участников найдется участник, выигравший у обоих. Какое наименьшее количество команд могло участвовать в таком турнире? Рис. 9 1 и 21 81 91 2 12 22 82 92 3 13 23 83 93 4 14 24 84 94 5 15 25 85 95 6 16 26 86 96 7 17 27 87 97 8 18 28 88 98 9 19 29 89 99 10 20 30 90 10С Рис. 10 211. Какое из двух чисел больше: у/3 -\-yfE или 212. В треугольнике ABC проведены медианы AM и BN, пересекающиеся в точке О. Найдите углы треугольника ABC, если периметры треугольников AN О и В МО равны, а около четырехугольника CNOM можно описать окружность. 29
История научных идей и открытий 213. Академик И.П.Павлов возле своей лаборатории установил бюст великого математика. Кто этот математик и в связи с чем физиолог Павлов установил его бюст? 214. Пифагорейцы не любили некое целое число за то, что оно расположено между двумя целыми числами, каждое из которых выражает площадь некоторого прямоугольника с целыми сторонами. Найдите это целое число. Что еще вы о нем знаете? 215. Математики в течение многих столетий искали генератор простых чисел, т.е. функцию f(n), значения которой - простые числа при всех натуральных п. В частности, Эйлер нашел несколько многочленов, дающих много простых чисел. Например, многочлен п2+п + 4\ имеет простые значения при всех 1 < п < 39. Однако Эйлер установил, что никакой многочлен не может быть генератором простых чисел. Докажите это. 216. Еще в V веке до новой эры Гиппократ Хиосский, пытаясь решить некоторую знаменитую гео- р ,, метрическую задачу, доказал, что площадь выделенных на рисунке 11 луночек равна площади прямоугольного треугольника ABC, а также, что сумма четырех площадей, выделенных на рисунке 12, равна площади трапеции ABCD. Какую задачу решал Гиппократ, когда и кем она была решена? Докажите утверждения Гиппократа о луночках. 217. Какие числа математики XV-XVI веков называли воображаемыми?
XI ОЛИМПИАДА Салоники (Греция), 2002 Письменный тур 218. Найдите: а) количество; 6) сумму цифр в десятичной записи числа 9-99-9999-...-99^ 22002 (количество цифр в каждом числе в 2 раза больше, чем в предыдущем). 219. В выпуклом четырехугольнике ABCD углы В и D прямые, а АВ = ВС. Найдите площадь четырехугольника А В CD, если расстояние от точки В до прямой AD равно А. 220. Решите уравнение (2х3+х-3)3 =3-х3 . 221. Может ли сумма квадратов трех последовательных натуральных чисел быть равной: а) кубу натурального числа; б) сумме кубов двух последовательных натуральных чисел; в) сумме кубов п > 3 последовательных натуральных чисел? 222. В треугольнике ABC проведены биссектрисы АК и BL. Найдите угол А, если известно, что KL - биссектриса угла АКС. 223. Груз общей массой 36 т представляет собой несколько ящиков. Масса каждого ящика не больше 1 т. За какое наименьшее количество рейсов заведомо можно перевезти этот груз на четырехтонном грузовике? 224. При каких тип клетчатый прямоугольник тхп можно разрезать на «уголки» из трех клеток? Устный тур 225. Из пунктов А и В одновременно навстречу друг другу выехали два автомобиля. Через сколько времени они встретились, если автомобиль, выехавший из А, доехал до пункта В через 9 часов после встречи, а автомобиль, выехавший из В, приехал в А через 4 часа после встречи? 226. Вершины четырехугольника ABCD лежат на сторонах четырехугольника AXBXCXDX. Может ли площадь ABCD быть больше площади AXBXCXDX ? 31
227. Является ли число 16016003 простым? 228. Сколько сторон у выпуклого я-уголышка, если число его диагоналей равно 119? 229. Какое наибольшее количество месяцев, содержащих 5 пятниц, может быть в году? 230. Даны прямая / и точка А вне ее. Проведя всего 3 линии циркулем и линейкой, постройте прямую, проходящую через точку А параллельно /. 231. Какое из положительных чисел а или Ь больше, если 232. В однокруговом турнире по волейболу (за победу присуждается одно очко, за поражение - ноль, ничьих не бывает) участвовали 12 команд. Можно ли выбрать три команды А, В и С так, чтобы А выиграла у В, В - у С, а С - у А, если ни одна из команд не набрала 7 очков? 233. Сколько сейчас лет моему племяннику, если в году с номером х2 ему исполнится х лет? 234. Три мотоциклиста А, В и С выехали из одной точки кольцевой дороги с постоянными скоростями в одном направлении. Через некоторое время они снова оказались в одной точке. Сколько раз мотоциклист А обгонял С, если А обгонял В 3 раза, а В обгонял С 4 раза? История научных идей и открытий 235. Легендарная школа Пифагора, заложившая основы математической науки, среди прочих задач занималась задачей о целочисленных прямоугольных треугольниках. В частности, пифагорейцы нашли бесконечные серии (не все) троек натуральных чисел (a, b, с), для которых а2 +Ь2 = с2. Вслед за пифагорейцами выясните, существует ли целочисленный прямоугольный треугольник, одним из катетов которого является число: а) 2001; б) 2k + 1, где k - произвольное натуральное число. 236. В древнем Египте представляли дроби в виде суммы различных долей (т.е. дробей вида — ). В папирусе жреца Ахмеса имелись даже таблицы таких представлений для дробей 2 вида — для 5 < п < 99 . Представьте в виде суммы долей дроби: п ^ 2 ал 7 ) б) 32
237. Архимед при вычислениях, связанных с окружностью, пользовался утверждением, которое в современной формулировке выглядит так: «В дугу А В вписана ломаная AM В из двух отрезков {AM > MB). Тогда основание перпендикуляра КН, опущенного из середины К дуги А В на отрезок AM, делит ломаную пополам: АН = НМ + MB». а) Докажите утверждение Архимеда, б) Какую тригонометрическую формулу заменяло в вычислениях Архимеда это утверждение? Запишите эту формулу. 238. Французский математик монах Марин Мерсенн (1588— 1648) состоял в переписке с крупнейшими математиками своего времени (Ферма, Паскалем, Декартом и др.). Его переписка исполняла роль своего рода математического журнала. Сам Мерсенн изучал, среди прочего, совершенные числа, т.е. числа, равные сумме своих делителей, отличных от самого числа. Совершенные числа связаны с простыми числами Мерсенна, т.е. с простыми числами вида 2п - 1: всякое четное совершенное число имеет вид 2Я~1 (2" -11, если 2" -1 простое число. Докажите, что если 2п -1 простое число, то и п простое. 239. 5 августа 2002 года исполнилось 200 лет со дня рождения великого математика, прожившего очень короткую жизнь - неполных 27 лет. Он внес гигантский вклад во многие разделы математики. Теоремы и термины, связанные с его именем, известны всем математикам, начиная с первого курса университета. Одним из самых его замечательных результатов является доказательство теоремы, окончательно решившей проблему, связанную с алгебраическими уравнениями и не поддававшуюся усилиям математиков в течение многих столетий. Кто этот математик и о какой теореме идет речь? Назовите какие-нибудь известные вам термины, теоремы и факты, связанные с его именем. 2 Библиотека Кваш
XII ОЛИМПИАДА Пущино, 2003 Письменный тур 240. Запишите в виде несократимой дроби число 1010111110101 110011111001Г 241. На продолжении биссектрисы угла С треугольника ABC за точку С взяли такую точку D, что ZADB = — Найдите CD, если ВС = а, АС = Ъ. 242. Решите уравнение 243. При каких натуральных п число: а) 5п + 4" ; 6) 5" - 4я является полным квадратом? 244. Три равные окружности проходят через точку Я. Пусть Ау Bt С - вторые точки пересечения этих окружностей. Найдите угол ВАС у если АН = ВС. 245. Депутат Госдумы по возвращении с Канарских островов, где он был в командировке, узнал, что его избрали председателем комитета Госдумы, причем за него было подано больше 68, но меньше 69 процентов голосов. Какое наименьшее количество депутатов могло участвовать в голосовании? 246. Пусть aXya2,...,am,bx,b2,...,bn - некоторые числа, причем ах+а2+... + ат =Ь|+Ь2+... + Ья. а) Можно ли вписать в клетки пустой таблицы m x n не более тп + п — 1 чисел так, чтобы суммы чисел в строках были равны ava2,...,am , а в столбцах bvb2,...,bn соответственно? б) Тот же вопрос, если все числа ava2i...,am,bvb2,...,bn положительны, а вписывать в таблицу следует неотрицательные числа. Устный тур 247. Группа туристов вышла из лагеря в 11 часов утра. В 11:30 один из туристов вспомнил, что он оставил в лагере компас, и побежал назад в лагерь. Взяв компас, он тут 34
же побежал обратно и догнал группу в 13 часов. В котором часу он прибежал в лагерь? 248. Можно ли разрезать произвольный выпуклый я-уголь- ник на равнобедренные треугольники? 249. Упростите выражение л/20032 + 20032 • 20042 + 20042 . 250. Существуют ли две трапеции такие, что боковые стороны каждой из них равны основаниям другой? 251. Найдите все натуральные числа, которые нельзя представить в виде суммы двух взаимно простых чисел, больших 1. 252. Можно ли из полосок 1x1, 1x3, 1x5, ... , 1х 2003 сложить прямоугольник, каждая сторона которого больше 1? 253. Разделите угол — на три равные части циркулем и линейкой. 254. Отец и сын катались по кругу на катке. Время от времени отец обгонял сына. Когда сын стал бегать в противоположном направлении, они стали встречаться в 5 раз чаще. Во сколько раз отец бегает быстрее сына? 255. На дне рождения доктор Ватсон угощал гостей треугольным тортом, который он разрезал по биссектрисам углов на б частей. Холмсу достался кусок в виде прямоугольного треугольника, после чего Холмс заявил, что кому-то еще из гостей тоже достался кусок в форме прямоугольного треугольника. Прав ли Холмс? 256. Сумма нескольких положительных чисел равна сумме их квадратов. Что больше: сумма четвертых степеней этих чисел или сумма их кубов? История научных идей и открытий 257. В 2003 году математическая общественность всего мира отметила 100-летие со дня рождения великого российского математика, академика А.Н.Колмогорова (1903-1987). Он был не только выдающимся ученым, но и педагогом, автором многих учебников, книг и статей для школьников и учителей. Что вы знаете о научной и педагогической деятельности А. Н. Колмогорова? 258. Николай Коперник (1473-1543), создавая свою гелиоцентрическую систему мира, изучал сложные движения тел. В частности, он рассмотрел и решил такую задачу. Окружность со радиуса 1, касающаяся изнутри неподвижной окружности радиуса 2, катится без скольжения по внешней 35
окружности. По каким траекториям будут при этом двигаться точки окружности со? Решите эту задачу и вы. 259. По праву считающийся одним из создателей теории чисел П.Ферма (1601-1665) полностью решил задачу о том, какие натуральные числа представляются в виде суммы двух квадратов натуральных чисел. Для этого ему, в частности, понадобилось следующее утверждение: если числа тип пред- ставимы в виде суммы двух квадратов, то и тп тоже представи- мо в таком виде. Докажите это. 260. Один из замечательных математиков древности Аполлоний Пергский (III-II в. до н.э.) доказал, что геометрическим местом точек, отношение расстояний от которых до двух данных точек постоянно и не равно единице, является некоторая окружность (она называется окружностью Аполлония). Дока- МА _ жите, что, в частности, все точки М, такие что = 2 , где MB АВ = 3, лежат на некоторой окружности. Найдите центр и радиус этой окружности. 261. Леонард Эйлер (1707-1783) доказал в 1752 году теорему, которая считается первой в истории науки теоремой одного из самых глубоких разделов математики - топологии. Теорема Эйлера гласит, что число вершин В, число ребер Р и число граней Г произвольного выпуклого многогранника удовлетворяют равенству Пользуясь теоремой Эйлера, докажите, что всякий выпуклый многогранник имеет либо треугольную грань, либо трехгранный угол (т.е. либо в какой-то вершине сходятся ровно 3 ребра, либо среди граней найдется треугольник). Подсказка: предположив противное, оцените двумя способами число ребер.
XIII ОЛИМПИАДА Сеч (Чехия), 2004 Письменный тур 262. Может ли число, оканчивающееся цифрами: а) 987654321; 6) 123456789 быть квадратом целого числа? 263. Биссектрисы внешних углов при вершинах В и С треугольника ABC пересекаются в точке О, причем ОВ = ОС. Найдите АВ, если /ЛОВ = а , а радиус описанной около треугольника ABC окружности равен R. 264. Решите уравнение 265. Последовательность {/"„} (числа Фибоначчи) удовлетворяет условиям /J = f2 = 1, fn = /"„_! + fn_2 при п > 2. Какие числа вида: a) 3k; 6) 2k содержатся в этой последовательности? 266. Из вершины треугольника проведен в точку на противоположной стороне отрезок, делящийся на 3 равные части точками его пересечения с вписанной окружностью. Может ли этот отрезок быть: а) высотой; б) медианой; в) биссектрисой треугольника? 267. Какое наименьшее значение может принимать сумма всевозможных попарных произведений п чисел, каждое из которых по модулю не больше 1? 268. Можно ли разрезать квадратный лист клетчатой бумаги (2п -1) х (2п - 1) на части вида Ц иЧ *-i, если: а) п = 2; б) п = = 3; в) п = 4; г) п > 5 - любое натуральное число? Устный тур 269. Из пункта А в пункт В вышел пешеход. Одновременно с ним из В в А выехал велосипедист. Через час пешеход оказался ровно посередине между А и велосипедистом. 37
Еще через 15 минут они встретились и продолжили свой путь. Сколько времени пешеход шел до В? (Скорости пешехода и велосипедиста постоянны.) 270. Может ли делиться на 30030 произведение двух натуральных чисел, сумма которых равна 30030? 271. Какое наименьшее количество «уголков» из трех клеток следует вырезать из шахматной доски 8x8 клеток, чтобы из оставшейся части доски нельзя было бы вырезать ни одного такого же уголка? 272. Найдите углы треугольника ЛВС, если его ортоцентр является точкой пересечения двух трисектрис углов В и С (трисектрисы - отрезки, делящие угол на три равные части). 273. Что больше: 5100 + б100 или 7100 ? 274. Можно ли записать в строку 17 чисел так, чтобы сумма любых пяти подряд стоящих чисел была отрицательна, а сумма всех 17 чисел положительна? 275. Делится ли число 2202 + 1 на 2201 + 251 +1 ? 276. На окружности расставлены п точек, занумерованных подряд числами 1, 2, 3, ..., п. Двое играют в следующую игру. Каждый по очереди проводит хорду, соединяющую точки с номерами одной четности. Каждая проведенная хорда не должна иметь общих точек (даже концов) с проведенными ранее. Побеждает тот, кто сделал последний ход. Кто выигрывает при правильной игре - начинающий или его партнер, если: а) п = 2004; 6) п = 2005? История научных идей и открытий 277'. Архимед, пытаясь решить задачу о квадратуре круга, рассмотрел арбелос - фигуру, ограниченную тремя полуокружностями (см. рис. 13). Найдите площадь арбелоса, если CD = a. 278. Великий немецкий математик К.Ф.Гаусс измерял углы треугольника, образованного вершинами трех удаленных друг от друга гор. Зачем он это делал? 279. Великий астроном древности, создатель геоцентрической системы мира Клавдий Птолемей был и выдающимся математиком. В частности, он доказал, что сумма произведений противоположных сторон вписанного четырехугольника равна произведению 38
его диагоналей. Какая формула тригонометрии стоит за этой теоремой? 280. П.Ферма впервые поставил вопрос о представимости простых чисел квадратичными формами, т.е. о возможности записать данное простое число р в виде р = ах2 + 2Ьху + су2, где а, Ь, с, х, у - целые числа. Докажите, что если р = х2 + ху + у2 , то р = 6& + 1 при некотором к. 281. Французский историк А.Вандаль писал: «... он слишком большой математик для того, чтобы судить о политических делах. Он ни рдного вопроса не рассматривал под настоящим углом зрения, он всюду искал хитроумных комбинаций, высказывал лишь математические идеи и вносил в администрацию дух бесконечно малых...». О каком великом математике и астрономе идет речь?
XIV ОЛИМПИАДА Ретимно (о.Крит, Греция), 2005 Письменный тур 282. Может ли число вида быть простым? 283. Вне правильного треугольника ЛВС, но внутри угла ВАС взята точка М, для которой ZCMA = 30° , ZBMA = 45° . Найдите ZABM . 284. Решите уравнение x^jy -1 + уу/х -1 = ;п/. 285. Можно ли разбить числа 1, 2, ..., 30 на группы: а) по 5 чисел; б) по 6 чисел так, чтобы суммы чисел во всех группах были одинаковы? 286. В треугольнике ABC проведены медиана AM, биссектриса AL и высота АН. Найдите радиус окружности, описанной около треугольника ABC, если AL = /, АН = h и AL является медианой треугольника МАН. 287. Пусть а > 1 ~ корень уравнения а3 - а -1 = 0 . Вычислите л/За2 - 4а + \/За2 + 4а + 2 . 288. Можно ли отметить на плоскости 225 точек так, чтобы наибольшее из расстояний между ними было не больше 21, а наименьшее - не меньше 3? Устный тур 289. Известно, что доля блондинов среди голубоглазых больше, чем доля блондинов среди всех людей. Что больше - доля голубоглазых среди блондинов или доля голубоглазых среди всех людей? 290. На гипотенузе прямоугольного треугольника найдите точку, для которой сумма квадратов расстояний до катетов минимальна. 291. Можно ли За4 + 1 (а - целое число) представить в виде суммы трех квадратов целых чисел? 292. Прямая /, перпендикулярная гипотенузе А В прямоугольного треугольника ABC, пересекает прямые АС и ВС в 40
точках Е и D соответственно. Найдите угол между прямыми AD и BE. 293. В семье шестеро детей. Пятеро из них, соответственно, на 2, б, 8, 12 и 14 лет старше младшего. Сколько лет младшему, если возрасты всех детей - простые числа? 294. Существуют ли на плоскости 6 точек такие, что любые три из них являются вершинами равнобедренного треугольника? 295. Найдите наименьшее значение функции six2 - 6х + 13 + у1х2 - 14* + 58 . 296. Первые цифры чисел 5" и 2" одинаковы. Каковы эти цифры? 297. Три грации, имевшие по одинаковому количеству плодов, встретили девять муз. Каждая из граций отдала каждой из муз по одинаковому числу плодов, после чего у каждой музы и каждой грации стало по равному числу плодов. Сколько плодов было у каждой грации, если всего плодов у них было не больше 70? 298. В выпуклом четырехугольнике ABCD точка К - середина стороны ВС, а площадь треугольника AKD равна половине площади всего четырехугольника. Найдите длину медианы КЕ треугольника AKD , если А В = а , CD = Ь. 299. Человек обычно приезжал на станцию одним и тем же поездом. К этому времени за ним обычно приходила машина и отвозила его домой. Однажды он приехал на 1 час раньше, пошел пешком, встретил по дороге машину и вернулся домой на 20 минут раньше обычного. Сколько времени он шел пешком? 300. Сколько решений имеет система уравнений х + у2 + z4 = 0, у + z2 + х4 = 0, z + x2 +уА =0? История научных идей и открытий 301. Назовите нескольких математиков - современников д'Артаньяна. 302. Знаменитый немецкий математик и философ Готфрид Вильгельм Лейбниц считал, что многочлен х4 + 1 нельзя разложить на 2 множителя с действительными коэффициентами. Согласны ли вы с ним? 303. В VII веке индийский математик Брахмагупта сумел 41
записать выражение VlO + л/24 + л/40 +V60 без «двухэтажных» радикалов. Проделайте это и вы. 304. В «Книге лемм», приписываемой Архимеду, имеется замечательное предложение: «Продолжи хорду АВ произвольного круга на отрезок ВС, равный радиусу, и проведи через С диаметр FDE. Тогда дуга АЕ будет втрое больше дуги BF». Докажите это предложение. 305. Леонард Эйлер доказал, что Также он нашел Найдите ее и вы 1+ * 4 + 22 4 сумму 1 + 32 "* 1 З2 +* 1 52 + * 1 ■■+я2+-- 1 - + (2л + 1)2 п2 6
ОТВЕТЫ, УКАЗАНИЯ, РЕШЕНИЯ 1. Поскольку 53 + 96 = 109 + 40 = 83 + 66 = 149, данное число делится на 149. 2. Исходное выражение равно 1 1 1 +1/(1 + х) + 2 + х ' где х - дробь в знаменателе второго слагаемого. 3. rf-Уз . Пусть О - центр, a R - радиус окружности, описанной около треугольника ABC, OE - перпендикуляр, опущенный из точки О на АВ. Воспользуйтесь подобием прямоугольных треугольников АОЕ и ADC. 4. х = v2 и х = пп , п е Z . Если слагаемые в левой части уравнения не равны 0, то они имеют одинаковые знаки. 5. Решений нет. Представим уравнение в виде 19(.r3-100) = Правая часть делится на 7, а левая - нет. Убедитесь в этом, рассматривая остатки от деления х3 на 7. 6. Сложите неравенства а > k + nb/a и Ъ > п + ka/b и воспользуйтесь неравенством между средним арифметическим и средним геометрическим. 7. 45°. На продолжении катета ВС за точку С (рис.14) отложим отрезок CF = BE = CD, а на перпендикуляре к АС в точке D отложим отрезок DG = DC. Четырехугольник DCFG - квадрат, треугольники ADG, BCD и EFG равны, BD \\ GE , AG = = EG, /AGE ~ прямой. Итак, ZAEG = 45° = ZBOE . 8. Все такие окружности проходят через точку (0; 1). Возможны 3 случая расположения параболы (рис.15). Во всех случаях ОА ОВ = ОСОР, откуда 43 Рис. 14
а) \xix2\ = \q\ • OP , а так как по теореме Виета ххх2 = q , получаем р=\. 9. Не могут. К каждому из квадратов примыкают ровно 4 квадрата, образующих «крест». Сумма всех чисел в этих 5 квадратах равна 17. Каждый квадрат принадлежит 5 крестам. Следовательно, упятеренная сумма всех вписанных чисел равна 17-54 , что невозможно, если все эти числа целые. 10. Пусть а - корень данного уравнения, т.е. а3 - За +1 = 0. Докажем, что х = а2 - 2 - тоже его корень. Для этого вычислим х3 - Зх +1 = {а2 - 2)3 - З[а2 - 2) + 1 = а6 -6ал + 9а2 -1. (* ) Поскольку а3 = За -1, то а6 = 9а2 - 6а + 1, а4 = За2 - а . Подставляя последние два выражения в (*), получаем, что х3 - Зх +1 = 0 . Утверждение доказано. Равенство а2 - 2 = а невозможно. Поэтому либо а2 - 2 = Ъ , либо а1 - 2 = с . Для завершения решения убедитесь, что равенство а2 - 2 = с невозможно. Поэтому а2-2 = Ь, Ь2-2 = с, с2-2 = а. 11. Пусть АХ,ВХ,СХ -точки пересечения прямых АЛ', ВВ' с СС со сторонами ВС, АС и АВ с прямыми ВСу АС и АВ соответственно (рис.16). Докажем, что АС а ВА< СВХ . пользуемся теоремой Чевы (она была доказана накануне тура на консультации для участников олимпиады).
Введем стандартные обозначения ВС = а, АС = Ъ, АВ = с, ZA = a, ZB = $, ZC = y. Пусть также ZC'BA = ZA'CB = = ZB'AC = ф . Рассмотрим площади треугольников 5t = SACC', 5, AG So ВА< 5, СЯ заметим,что - = — ,- = — ;- = — Вычислим теперь, например, 5t: 1 1 be 5t = AC- ACsin(a + (p) = sin(a 2 4 соБф Аналогично вычисляются и остальные площади S{. Нетрудно видеть, что с с с АС В А С В с* с* с* ' ' ' С R А С R А 12. Пусть хх < х2 < х3 < х4 - данные положительные числа. Можно считать, что они попарно различны. Воспользуемся тем, что х-у _ (1 1 + х + у + 2ху (1 + 1/х)(1 где a = arctg (I + \/у), Р = arctg (1 + \/х), причем числа аир принадлежат интервалу (к/4; к/2), так что среди чисел arctg(l + l/j:1), г =1, 2, 3, 4, найдутся два, разность которых меньше, чем л/12 (tg л/12 = 2 - V3 ). 13. Второе. Пусть я = 1991. Тогда (у/а-1 + VflTT)2 = 2а + 2л/я2 -1 < 4д . 14. Докажите, что треугольник ABC - равнобедренный (АС = ВС). 15. -р -q. Заметим, что Sk = k(ak + /), где а и / - некоторые числа. По условию Sp = q , Sq =p, т.е. p(ap + l)= q} q(aq + l) = p. Вычтем из верхнего равенства нижнее, получим после сокращения на р - q, что а (р + q) + / = -1 и, наконец, Р + ?) + 0 = -Р ~ Я . 45
16. Пусть ЕГ - точка на прямой ЛВ, симметричная точке В относительно точки Л. Искомое ГМТ состоит из двух дуг двух окружностей радиусом АВ, проходящих через точки Ли В' , за исключением точек Л и В' (рис.17). Рис. 17 Рис. 18 Пусть ABC - треугольник, удовлетворяющий условию задачи (рис.18). Проведем через точку В прямую, параллельную AM. Пусть К - точка пересечения этой прямой с АС. Угол ВКН равен 30°, так как из условия следует, что гипотенуза В К в 2 раза больше катета ВН. Таким образом, точка К лежит на «сегменте, вмещающем угол в 30° и опирающемся на отрезок ЛБ». Точка С при этом симметрична точке К относительно точки Л. Обратное утверждение докажите самостоятельно. 17. Из условия следует, что а3 + Ь3 = 1 - (1 - а - bf т.е. b)( a2 - Если а + b = 0, то с = 1. Если а Ф -Ь , то из полученного равенства получится, что (д-1)(Ь-1) = 0. 18. Шестнадцать, когда каждая сторона одного из четырехугольников пересекается с каждой стороной другого (рис.19).
19. При п < 5 • Неравенство приводится к виду Если п < 5, неравенство справедливо всегда. Если же п > 5, оно неверно, например, при хх - х2 = ... ... = хя_1 = 1, *„ = 2. 20. Нельзя. Заметим, что сумма i о <nm 1991 1992 1 + 2 + ... + 1991 = четна. Пусть есть какой-то набор целых чисел с суммой S. Возьмем несколько из них с суммой 5а (а - целое) и запишем вместо них число а. Новая сумма равна S - 5а + а = S - 4а. Мы видим, что четность суммы сохраняется. Исходная же сумма четна. Отсюда и следует ответ. 21. \[\0 при х = у = z = - . Дан- ная сумма - это длина ломаной линии, помещенной на рисунке 20 (АВ = Рис. 20 = 1, ВС = 3). Минимум суммы достигается, когда х = у = z, a ломаная при этом совпадает с отрезком АС. 22. а) Пользуясь теоремами о вписанных углах и углах между касательными и хордами (рис.21,я), получим, что Zl = 180° -ZSAT = \80°--, т.е. ZX + Z2 = - . Рис. 21 47
Далее, ZPTS = ZPST = 18°° a = 90° -1. Но тогда ZBTC = = 180° - f90° - -1 - Z\ - Z2 = 90° . Итак, ВС - диаметр. Q 6) Аналогично предыдущему, ZPAT = ZPTA = 90° - - = = ZCAH (рис.21,б), а поскольку ВС - диаметр, Z2 - ZHCA = = 180° - ZPTA - 90° = ^ . Отсюда следует, что ZP#C = 90° • 23. Второе. Пусть а = 1992. Тогда (а +1)-1 (* -1)'+1 = ^\(а2 - if < (а2 - if < *2* . 24. Пусть w = 1992. Разобьем данную сумму на 3 группы по п слагаемых: 1 11 11 1 Sn = + + ++ + ++ + 1 ++ ... + ++ ... + 2w 2w + l 3w 3w + l An Поскольку 11 \ n - < + ...4- <■ 2 п + 1 2я п + \> I 1 JL п 3 2я + 1 '" Зя 2я + Г II 1 я 4 Зя +1 4я Зя + Г получаем 111 я я я 11 2 + 3 + 4< п < яП" + 2я + 1 + Зя + 1 К + 2 + 3' 13 . 11 т.е. — < Sn < — . 25. - - : 2, — : -= . Пусть д = х2 - х - 1, Ь-2х2- 3 2 4 - х - 7 . Тогда я3 + б3 = (я + bf , откуда яЬ(д + Ь) = 0. Осталось решить уравнения а - О, Ь = 0, я + b = 0. 26. 2. Пусть ЛС = Ь, АВ = с, ВМ = тс, CN = ть. Так как 48
четырехугольник AMON описанный, имеем - + -^- - - + -^-, Z* ij Z, Cj откуда = . Из последнего равенства следует, что при с > Ь будет тс > ть и, наоборот, тс < ть при с < Ъ. Но в любом треугольнике большей стороне соответствует меньшая медиана. Поэтому при Ь Ф с равенство невозможно. 27. а) Нет. Сумма любых десяти последовательных квадратов может быть записана так (п - натуральное число): Sn=(n-4)2 + (п - З)2 +... + п2 + (п + I)2 +... + (п + 5)2 = Докажите, что выражение в скобках не делится на 5 ни при каких п. 6) Например, 182 +192 +... + 282 = 772. Сумма одиннадцати квадратов равна Если Sn - квадрат, то я2+10 делится на 11, причем п должно быть нечетным, т.е. п = 22& ± 1. При п = 23 имеем ll(232 + 10) = 11-539 = 772. 28. Пусть а - корень уравнения f (х) = х2 + рх + q = 0 , а Р - корень уравнения д (х) = х2 - рх - q - 0 , тогда а2 - 2ра - 2q = За2 - (2а2 + 2ра + 2q) = За2, a р2 - 2рр - 2q = -р2 + (2р2 - 2^р - 2q) = -р2 . Это значит, что квадратный трехчлен х2 - 2рх - 2q принимает на концах отрезка [а; Р] значения разных знаков, т.е. имеет корень на интервале (а; Р). 29. Пусть (для определенности) ZBDE - острый, ZA - а , ZB = p, ZC = y (рис.22), тогда ZBDE = а + — и точка Е лежит на продолжении АС за точку С, BE = = DE. Пусть F - точка пересечения КЕ с АВ. Очевидно, что DF = Рис. 22 В 49
= FB, причем DF || ВС . Но тогда ZDBE = ZBDE = n - у - - , a ZCBF = я-у=а- Тем самым треугольники ВЕС и ЛЕЯ подобны, т.е. — = — , BE2 = СЕ • АЕ , a BE = D£. СЕ .Bfi 30. Числа /W и п делятся на 4. Предположим, что в результате допустимых операций возникла таблица с «шахматным» расположением единиц и минус единиц. Возьмем какую-либо строчку и рассмотрим ее «предысторию». При каждой смене знака с нашей строчкой либо ничего не происходит, либо меняются знаки у некоторых двух соседних чисел. Пусть а - количество единиц, а Ъ - количество минус единиц в этой строчке. Если оба числа были единицами, то в новой строчке окажутся а' = а -2 единиц и Ъ' - Ъ + 2 минус единиц. Если оба были минус единицами, то а' = а + 2 , Ъ' = Ъ - 2 . И, наконец, если это были числа разных знаков, то а' - а , Ъ' = Ъ . Во всех случаях а - Ъ и а' -Ъ' дают одинаковые остатки при делении на 4 ( а' -Ь' = a -b(mod4) ). Допустим, что п нечетно и верхняя строка «шахматной» таблицы начинается с +1, т.е. имеет вид +1, -1, +1, -1... В исходной таблице а = п, Ъ = 0, в «шахматной» а'-Ъ' - 1. Поэтому я-Osfl'- -fe/(mod4), т.е. п s I(mod4) и п = 4k + 1. Аналогично, если первая строка «шахматной» таблицы начинается с -1, то п = = 4/ + 3. (Для этой строки а' - Ь' = -1.) 1 1 1 1 1 1 1 1 1 1 1 1 1 1 1 1 1 1 1 1 -1 -1 1 1 1 1 1 1 -1 -1 1 1 50 1 -1 -1 1 -1 1 -1 1 1 -1 -1 1 -1 1 -1 1 V 1 -1 1 -1 -1 1 1 -1 1 -1 -1 1 -1 1 -1 1 V 1 -1 1 -1 -1 1 -1 1 1 -1 1 -1 -1 1 -1 1 Рис. 23
В обоих случаях вторая строка получается из первой сменой знака у всех чисел, т.е. одновременно должно быть п = 4k + 1 и п = 4/ 4- 3 , что невозможно. Итак, п - четно. Аналогично, четным должно быть и т (в соответствующем рассуждении строки нужно заменить столбцами). Пусть теперь п = 4k + 2, и «шахматная» таблица начинается с +1. Для нее в начале а = 4k +2, Ъ = 0, в конце а' -Ъ'. Но числа аи а' -Ъ' дают разные остатки при делении на 4. Тем самым числа пит обязаны делиться на 4. Таблицу же 4k x 41 следует разделить на квадраты 4x4, а в каждом из этих квадратов получить «шахматную» раскраску. Это делается без труда (рис.23). 31. Не хватит. В конце 7-го дня у него остается — часть 4 о мыла, а его хватит только на 1 день. 32. 6. Для того, чтобы вырезать центральный кубик, нужно не меньше 6 разрезов. 33. Могут. Рассмотрим равнобедренный треугольник с очень большим основанием и очень маленькой высотой. 34. 90°. Заметим, что 92 +82 = I2 +122 . Пусть а, Ь, с, d - последовательные стороны четырехугольника ABCD (рис.24), причем а2 + с2 = Ь2 + d2 . По теореме косинусов а2 = х2 + + у2 -2xycosa, b2 = у2 + z2 +2*/zcosa , с2 = z2 +t2 -2ztcosa - 2zt cos a , d2 = x2 +12 + 2xt cos a . Из равенства сумм квадратов противоположных сторон легко следует, что cos a = 0. Рис. 24 Рис. 25 35. а) 16. В каждом квадрате 2x2 может находиться не более одного короля. Нужная расстановка показана на рисунке 25. 51
Рис. 26 6) 32. Нужно поставить коней на белые (либо на черные) поля доски. 36. Существует. Это прямоугольный треугольник с отношением катетов, равным V2 . 37. Восьмерками. Если 2" = ...ааа , то число ааа делится на 8 при п > 3. 38. Второе число больше. Преобразуйте разность данных чисел. 39. Это сумма первых десяти членов последовательности 31 + 35Де при k = = 0, 1, ...,9. 40. Существуют. На рисунке 26 это пятиугольники ABCDEA и ACEBDA. 41. 7744=88 . Пусть А = ааЪЪ - искомое число. Тогда А = 11(100я + Ь). Докажите, что Ъ = 4. Далее воспользуйтесь тем, что число 100я + 4 делится на 11, т.е. а = 7. 42. Можно. В первом столбце таблицы 9x9 ставим числа (сверху вниз) 1, 2, ..., 9, во втором 2, 3, ..., 9, 1, в третьем 3, 4, ..., 9, 1, 2, ..., в девятом 9, 8, ..., 1. Затем ко всем числам второго столбца прибавляем 9, к числам третьего - 18, ..., к числам девятого - 72. 43. 45°. На продолжении ВС за точку В возьмем точку N' такую, что BN' = DN (рис.27). Треугольники N'AM и MAN равны, a AM - биссектриса прямого угла N'AM . D Рис. 27 Заметим, что возможно лишь при В xk=2kz = 1, 2, п. Рис. 28 = — , причем равенство х - k2 = k2, т.е. х = 2k2. 45. %/2 . Пусть DK - основание перпендикуляра, опущенного на ВС из точки D, АС = х, BD = = а (рис.28). Тогда KD = = а/2, треугольники DC К и ABC подобны и а = 2/х. Осталось применить теорему косинусов к треугольнику ABD. 52
46. а) Достаточно заплатить 1 р, получив при этом сдачу 1 = 2 48-15-4-20 . 6) N = 218. Рассмотрите последние цифры уплачиваемых сумм и докажите, что N = 217 - наибольшая сумма, которую нельзя уплатить имеющимися монетами. 47. Пусть а = 20. Поскольку а1 + а2 +1 = а1 - а + а2 + а + 1 = [а2 + а + Ща3 + \)[а2 - а) +1), данное число является составным. 48. Воспользуемся методом индукции. При п = 1 неравенство, очевидно, справедливо. Пусть ах +... + дя ~ 1 + 2 + ... + я Преобразуем разность: Перепишем числитель полученной дроби: По предположению индукции 3(Д12+ ... +а2) >(2п+ •[)(<*,+... + а„). Поэтому А > За2+1 - (2п + ЗК+, - 2(в, +... + ап). Но из условия следует, что ап > п и Следовательно, А > За2+1 - (2ап + 3) ап+1 - ап [ап +1) = = (Зая+,+ап)(ап+,-ап-1)^0, так как яп+1 > д„ +1. Утверждение доказано. 53
49. 1. Для любого а сумма а + (я + \)-(а + 2) -(а 4-3) = = -4 . Поэтому данная сумма равна -4 • 498 +1993 = 1. *!!■ 51. 3994. После замены t = x2+2x получаем уравнение t2 -1993£ + 1995 = 0 . Пусть tx и t2 - корни этого уравнения. Пользуясь теоремой Виета, подсчитайте суммы квадратов корней уравнений х2 + 2х = tx и х2 + 2х = t2. 52. Нет. Такое число дает при делении на 8 остаток 5. Однако квадраты нечетных чисел дают при делении на 8 остаток 1. 53. Может. Если Smn - сумма нечетных чисел от т до я, то (п-т + \)(т + п) ._ . . Smn- 1 — . Можно подобрать тип так, чтобы выполнялись равенства т - п + 1 = 1993, т + п = 2 • 19931992. 54. V2 . Пусть ЛВ = х, BD = у, CD = z (рис.29). Тогда x+y+z<2, abcd - ^ - <-y(x + z), т.е. (x + z)y>\. Поскольку x + z <2- у , приходим к неравенству (2 - у) у > 1, или [у - 1) < 0 . Но это значит, что у=\,х + г=\,и все неравенства являются на самом деле равенствами. Поэтому стороны АВ и CD перпендикулярны диагонали BD, после чего без труда находим АС (рис.30). 55. Второе число. Это следует из соотношений \2 / / \2 /о\100 / л \100 56. Точки отрезка А В на плоскости хОу, где А = (0; 1), В = = (1; 0). Данное выражение - сум- ма расстояний от точки (х; у) до точек А и В. 54
57. Можно. Если 0 < q < 1 - длина меньшей стороны самого большого прямоугольника, мы должны разбить прямоугольник со сторонами 1 - q и 1 на q два прямоугольника с отношением сторон, равным q (рис.31). Из сказанного следует, что + q(1 - q) = 1, т.е. \-q q - корень уравнения q3 - q2 + 2q - -1 = 0. Это уравнение (неважно, что ис' мы не умеем его решать!) имеет корень на промежутке (0; 1). 58. . Разложите левую часть уравнения на множители: А + 8х - 7 = хА + 2х2 +1 - 2х2 + 8* - 8 = [х2 + l)2 - 2(х - 2)2 = 59. 753. Если десятичная запись куба числа оканчивается цифрой 7, то само число оканчивается на 3, т.е. имеет вид \0Л + + 3, где Л - некоторое двузначное число. Чтобы число (ЮЛ + З)3 оканчивалось на 2 семерки, нужно, чтобы на 2 семерки оканчивалось число 270Л + 27, т.е. чтобы А оканчивалось на 5. Аналогично находится и первая цифра числа А. 2 60. -S. Пусть К, L, М и N - середины сторон четырехугольника ABCD, а К', U , М' и N' - точки пересечения медиан треугольников ABS, BSCy CSD и DSA (рис.32). Четырехугольник K'L'M'N' получается из четырехугольника KLMN с помощью гомотетии с центром в точке 5 и 2 коэффициентом . Его пло- о щадь, следовательно, равна 4 S • Далее используйрис, 32 те то, что KLMN - параллелограмм, площадь кото- 55
рого равна — (это утверждение известно как теорема Варинь- она). 61. х = у = z = 0; х = у = z = +V2/4 . Вычитая из первого уравнения второе, получаем (х-z)(a2 + ab + b2) = z-х , где а = х + у, b = у + z. Отсюда следует, что либо х = у, либо а2 + ab + Ь2 = -1, но последнее равенство невозможно. 62. а) Нет, так как f(2) = д(2) = 6 , а 2 не делится на 6. б) Нет, так как /"(1/2) = д(\/2) = 1. в) Да. x = {f-g)2+2g-3f. 63. (a + b)/2. Докажите, что АА'В'В - трапеция, в которой ОО' - средняя линия. 64. а) Нет. Четность числа минус единиц на контуре пятиугольника не меняется, б) Нет. Произведение 9 чисел, расставленных в точках, закрашенных на рисунке 33, не меняется. 65. 3975. Докажите, что f(3k) = 3f(k), f(\) = 2 , f(2) = 3 . Вообще, f(n) = n + 3k, если 3k<n<2-3k, f{n) = 3n-3k+\ 3 мин 2 мин 1 мин 3 мин если 2 • 3* < п < 3*+1. ХЕ 66. На рисунке 34 показано, в какие моменты нужно переворачивать песочные часы. Одновременно уста- Рис. 33 2 мин 1мин Змин I I I 3 D 5 мин Рис. 34 навливаем и те и другие часы. По прошествии трех минут переворачиваем трехминутные часы, а в момент «остановки» пятиминутных часов кладем яйцо в кастрюльку, после «остановки» трехминутных часов еще раз их переворачиваем. 67. Можно провести 3 линии. Например, так (рис.35): берем точку О вне прямой /, проводим окружность радиусом ОР (1-я линия), затем проводим прямую Р'О , а потом и РК. 56
68. Не могут. Может быть, директор - жулик, однако в данном случае он прав. Это показывает следующий пример: 1; 1; 1; 1; -4,5; 1; 1; 1; 1; -4,5; 1; 1. Здесь показаны доходы и расходы фирмы по месяцам. 69. Пусть f (х) = х2 + рх + q . Предположим, что неравенство f{x) < 0 выполняется при некоторых х. Это равносильно тому, что /■(-£]< 0, т.е. р2 - Aq > 0 , q < ^- . Если -^ \ = т1 \ 2) 4 |_ 2J то т <-— < тю +1 , но тогда р = -2т - 1 и оба корня хх и х2 уравнения f (х) = 0 расположены на промежутке (т; т +1). Поскольку 1^ — jc2| < 1 , имеем х2 - 2ххх2 + х\ < 1 , т.е. 9 V2 -1 V2 -1 Р2 /Г - 4<7 < 1, или г—— < q . Итак, *—-— < q < *-- , т.е. 4 4 4 2 2 1 m+m<q<m+m + -i 4 что при целом q невозможно. 70. 1025. Для двузначных чисел КР и У Г введем обозначения КР = а, У Г = Ъ. По условию (\00а + bf = 105а + ЮО/w + Ъ , ( * ) где w - не более чем трехзначное число, обозначаемое ***. Отсюда видно, что b2 -b делится на 100, что возможно лишь при Ъ = 25 и Ъ = 76. Кроме того, из равенства ( * ) следует, ЮОООя2 < 105 (а +1), или а2 - \0а - 10 < 0 , а < 5 + V35 < 11. Итак, а < 11, но а двузначно и потому а = 10. Итак, либо КРУГ = 1025, либо КРУГ = 1076. Проверка показывает, что 1025 годится, а 1076 - нет. 3 71. ~7аЬ - Так как 36), SABL=—SABC, SALM = 3 л м = sabl - Кроме того, SABLM = pUc. 36 57
1 2 1 = -ab . Поэтому ~SABC =-ab . 2 о z 72. Параллелограмм. Рассмотрим параллелограмм BDC'B' (рис.37), где DC' = AC, В'С = BD . Периметр четыреху- гольника ABCD равен сумме от- резков DC, ЯС, В'С и СС . Он будет минимален, когда точка С рис 37 ^ совпадает с точкой пересечения диагоналей параллелограмма DBB'C , но тогда сам четырехугольник - параллелограмм. 73. Одно. Пусть, для определенности, х < у < z. Введем обозначения ot = х + у + z , о2 = ху + .rz + yz , o3 = ;п/2 . Числа .г, г/ и z - корни многочлена P(t) = (t-x){t -1/)(^ -z) = t3 -ott2 + a2t-a3. По условию a3 > 1, поэтому z > 1 и at > 1. Многочлен Р (t) принимает положительные значения при x<t<ynt>z. При этом если Р(1) < 0 , то у < 1, если же Р(1) > 0 , то х < 1 < г/. Далее, Р (1) = 1 - о, + о2 - о3 > 1 - о, + ata3 - с3 = (1 - о,) (1 - о3) > 0. Поэтому имеет место второй случай, т.е. 0<x<\,\<y<z. 74. 1, 7, 3, 2, 1, 1, 1, 2, 1, 1, и 1, 11, 2, 111111. 75. а) Не существует, так как число 1994 при делении на 9 дает остаток 5. б) Существует. Для числа вида п = 9k + 4 сумма цифр числа (10* - 3) в точности равна п. Выбор такого числа обусловлен сравнительной простотой вычисления суммы цифр: (10* - З)2 = 102* - 6 • 10* + 9 = 99^.9400^9 . Ясно, что сумма цифр равна 9k + 4. Для п = 1993 число k = 221. 76. Ярким примером является теория конических сечений, разработанная в деталях древними греками и нашедшая важнейшие приложения в XVII веке в трудах Кеплера и Ньютона по небесной механике. 77. Это П.Л.Чебышёв (1821-1894). Речь идет о его работах по так называемой конструктивной теории функций, изучающей способы измерения функций, связанные с их приближени- 58
ями более простыми функциями. Колоссальную роль играли также его работы в области теории чисел, теории вероятностей и математического анализа. 78. Это американский математик Норберт Винер, ставший творцом области знаний, называемой сейчас «кибернетика». 79. По формуле Кардано сложное выражение является корнем уравнения х3 + рх + а = 0 . Однако при 3 2 — + — <0 формула Кардано «отказывается работать». Для 27 4 того чтобы осмыслить этот феномен, и понадобились мнимые числа, позволяющее извлекать квадратные корни из отрицательных чисел. 80. Ал-Хорезми (787-850). Общеизвестный термин «алгоритм» является калькой с его прозвища, означающего «из Хорезма». Его труд «Краткая книга восполнения и противопоставления» («Китаб мухтасар аль-джебр ва-л-мухабала») положил начало алгебре как самостоятельному разделу математики. Да и само название «алгебра» восходит к названию операции «аль-джабр», состоящей в перенесении членов из одной стороны уравнения в другую. 81. К.Ф.Гаусс (1777-1855). «Rex Mathematicorum» - король математиков. На его надгробии изображен правильный 17-угольник, так как именно Гаусс впервые выяснил, какие правильные ЛГ-угольники могут быть построены циркулем и линейкой. 82. 18я. Воспользуйтесь тем, что = (10я -1)3 = 103" - 3 • \02п + 3 • 10" -1 = = 9^970^02^9. п-1 л-1 п (п п п\ (% 2л Ап\ _ 83. —; —; — и —; —; — . Пусть сначала треугольник \Э о о) у] 7 1 ) ЛВС остроугольный. Найдем углы треугольника АХВХСХ , образованного основаниями его высот (рис.38). Если ZA = а , то АСАХВХ = ZQAXB = ее и at = Z.CKAXBX - п - 2a . Аналогично, /ЛХВХСХ = п - 2р , ZAXCXBX = п - 2р . Если треугольники ABC и подобны, то (для определенности считаем, что a < р < у) 59
у = п-2а , Р = тс — 2p , a = л - 2y , откуда a = P = у = - . Для тупоугольного треугольника ЛВС (a < Р < у, Y>^) точки A, Av Bv В лежат на одной окружности (рис.39) (почему?) и В Рис. 39 потому ZBXAXB = a . Но точки АХ,С,СХ, А тоже лежат на одной окружности, так что ZCAXCX - /Л = a . Итак, ZBXAXCX = 2a , аналогично /ЛХВХСХ = 2р , а ZAXCXBX = к - 2 (а + Р) = п - - 2(п - у) = 2у - п . Осталось решить систему 2а = р , 2р = у, а = 2у-п. 84. а) 345; б) 165. Пусть ах < а2 < ... < ахо - средние числа в пятерках, упорядоченные по возрастанию. Докажем, что ak > 3k . В самом деле, числа ab...yak_x меньше, чем ak. С каждым из них меньшими ak заведомо будут еще по 2 числа из их пятерок, так что всего набирается 3 (Дг — 1) чисел, меньших ak . Но еще есть два числа, меньших ak, из пятерки, в которую ak входит. Итак, есть 3k - 1 число, меньшее ak. Но это значит, что ak>3k. Аналогично, есть 10 - k чисел ak+x,...,axo , больших ak. Вместе с каждым из них есть еще по 2 числа из их пятерок и, кроме того, еще 2 числа из k-й пятерки. Итого, заведомо есть 3(10- k) + 2 чисел, больших ak. Но это значит, что ak < 50 - 3(10- k) - 2 , т.е. ak <3& + 18. Разбиение на пятерки, для которых найденные оценки достигаются, легко построить. В них средними числами будут 21, 24, 27, ..., 48 и, соответственно 3, 6, 8, ..., 30. 85. (2; 1), (1; -1). Первое решение. Умножьте первое уравнение на у, а второе на л: и сложите полученные уравнения. Второе решение (для тех, кто знаком с комплексными числами). Умножив второе уравнение на г и сложив с первым, придем после преобразований к уравнению относительно 60
z = x + iy. 22-3z + 3-i = 0, корни которого zt=2 86. а) 50; 6) . Уравнение х2 - рх - q = 0 имеет положительный корень, не больший w, тогда и только тогда, когда п2 - рп- q > 0 . Осталось подсчитать количество точек с натуральными координатами (р; q) в треугольнике р > 1, q > 1, pn + q <n2. Если 1 < р < я - 1, то 1 < q < п2 - рп . Таким образом, интересующее нас число равно сумме ...+ п2 -р(п-\) + ... + п2 -(п-\)п = п2 -п п2 - 87. 5. Докажем, что ABCD - параллелограмм (рис.40). Пусть АВ = а , ВС = В , LM = с , DA - d , тогда а + В + с + + d = 0. По условию, —ал-—о- -1- 4 4 = XL, MN = -c+-d . 4 4 3 . . 3 1 - 4 Так как KL = -MN , - а + - Ъ = 4 4 4 4 4Д + 4 откуда — = —, т.е. а = -с , но это и значит, что ABCD - параллелограмм. Более того, ABCD - прямоугольник. Пусть 3 5 - его площадь. Тогда SKBL = SLCM = SMDN = SNAK = — S dZ 3 и 5 = 30 + -5 , т.е. 5 = 48. Но тогда ВС = 8 и 2R = 10, т.е. 8 R = 5. 88. тп делится на 8, т > 1, п >1. Допустим, что нужное нам разрезание возможно. Ясно, что тп делится на 4. Докажем, что на самом деле тп делится на 8. Если тп не делится на 8, одно из чисел т и п не делится на 4. Пусть это т. Рассмотрим «полосатую» раскраску нашего листа. На рисунке 41 горизон- 61
4 k + 2 I 1 r \ m- \ Pwc. 4/ тальные полосы покрашены через одну, начиная с верхней полосы. При т - 4k + 1 «полосатая» раскраска тоже начинается с верхней полосы. Если же т = 4k + 3, красим горизонтальные полосы через одну, начиная со второй. Во всех случаях получается нечетное число черных полос. Каждая буква Г будет содержать нечетное число черных клеток тп (одну или три). Если число —г нечетно, то количество закрашенных клеток тоже нечетно. Но это не так. Таким образом, тп делится на 8. Если т = 2Де, п = 4/, то лист может быть разрезан на прямоугольники 2x4, а каждый из прямо- 3 о т о тхп = 2 х4 Рис. 42 81 Рис. 43 угольников - на буквы Г (рис.42). Если т = 2k + 1, п = 8/, прямоугольник может быть разрезан на 2 прямоугольника 3x8/ и (т-3)хп (рис.43). Осталось разрезать прямоугольник 3x8 (см. рис.44). 89. Можно. Достаточно взять любое число окружностей, попарно касающихся друг друга в одной точке. 90. а) Нельзя. В сумму будет входить число ± - . Если сумма всех тхп=ЗхН Рис. 44 62 остальных чисел —, то — ± - = - , 67 8 5
где S делится на 8, г - нечетно (число q содержит двойку не более чем во второй степени). 6) Следует выкинуть все дроби, знаменатели которых делятся на 5, 7, 8, 9, 11, т.е. 6 чисел. Для оставшихся чисел требуемая расстановка знаков возможна: 1 1 1 — н— 4 6 91. Минутная стрелка впервые «догонит» часовую в — часа пополуночи. В этот момент дополнительная стрелка будет продолжением совпавших часовой и минутной стрелок. Еще через 12 „ часа все три стрелки снова совпадут, и т.д. От первого момента совпадения всех трех стрелок до следующего проходит часа. В сутки укладывается в точности 11 таких промежутков. За каждый из них дополнительная стрелка поворачивается 2 на 1 полный оборот и, кроме того, еще на полного оборота. Так что всего она совершит 13 оборотов. 92. Второе. Пусть х = Ясно, что у > х. Но тогда 1 у = ■ 1 1 1995 1995 2 + 1 3 + х 2 + - 3+г/ 93. 8. Сумма углов выпуклого 17-угольника 15л, а выпуклого четырехугольника 2л. Поэтому количество четырехугольников не мень- 15л 15 с д ше чем = — , т.е. не меньше 8. А на 2л 2 8 четырехугольников разрезать можно. Для этого надо сначала отрезать от 17- угольника пятиугольник, разрезать его на 2 четырехугольника (рис.45), а оставшиеся 14-угольники разрезать на 6 четырехугольников диагоналями. Рис. 45 63
94. По формуле для суммы членов арифметической прогрессии Sk = k = k(ka + b), где а и b - некоторые числа. Из равенства Sm = Sn следует, что вершина параболы f(k) = k (ka + b) - это точка к = . Один из корней уравнения /* (Дг) = 0 есть k = 0. Второй же корень симметричен точке 0 относительно вершины, т.е. равен т + п. Итак, sm+n = о. 95. х = у = ±-т= . Переписав уравнение в виде получим равносильную систему 96. tf = ±(4 + Vl5). Пусть я + л/15=т, —>/l5 = я , — = я + у/\5 . Перемножив полученные равенства, приходим после упрощений к уравнению 16 = тп + (т - n)\f\5 . Так как VT5 - иррациональное число, то т = п, тп - 16. 97. 2а. Пусть (для определенности) точка Е лежит на продолжении Л С за точку А (рис.46), а К - середина ЕС. Тогда DK = = КС, ZKDC = ZDCB, т.е. DK || ВС . Поэтому ZDKA = = ABC A = ZBAC и треугольник ADK равнобедренный. Но ЕС = = 2DK = 2AD = 2а. 98. Нет. Пусть (6, с, м) - количества белых, серых и малиновых хамелеонов в данный момент времени. При встрече разноцветных хамелеонов (скажем, белого и малинового) тройка чисел (6, с, м) переходит в тройку (6 - 1, с + 2, м - 1). Поскольку (6 - 1) - (с + 2) = б - с - 3, остаток от деления на 3 64 А Рис. 46 К
разности б - с не меняется, т.е. остается равным 1. Однако если бы все хамелеоны стали одноцветными, т.е. если бы в итоге получилась одна из троек (45, 0, 0), (0, 45, 0) или (0, 0, 45), соответствующая разность делилась бы на 3. 99. См. рисунок 47 а, б. а) б) а-Ъ а-Ъ а-Ъ а< а< N а-Ъ а-Ъ * Ъ Рис. 47 а) (а-Ъ)2 = а2-Ъ2 -2(а-Ъ)Ъ\ б) a2-b2 =(a-b)(a + b). 100. Сейчас это у/а . Автор обозначения - Р.Декарт. 101. Перечисленные ученые были директорами обсерваторий, и учили они астрономии. 102. Нуль мошенники легко переделывали в шестерку или девятку. 103. Не случайно К - математик. К нему приходили в гости братья N - натуральные числа. События, описанные в условии, происходили так. В первый раз второй брат взял шляпу первого, третий - второго и т.д. Во второй раз 1-й взял шляпу 2-го, 2-й - 4-го, 3-й - 6-го и т.д. На вешалке остались шляпы 1, 3, 5,... братьев. О том, что произошло в третий раз, подумайте сами. Описанные ситуации типичны для бесконечных множеств - каждое из них содержит подмножества, имеющие столько же элементов, сколько содержится в самом множестве. Первооткрывателем теории множеств был Георг Кантор. 104. Полученное число больше. Поскольку — = 0,0(714285), 14 а длина периода дроби равна 6, на 1996-м месте после запятой стоит цифра 2, а после ее вычеркивания на ее месте оказывается цифра 8. 3 Библиотека Кват 65
С" / А Я1 Ъ + с-а 105. ~ . Проведем через вершину А прямую /, параллельную ВС у и продолжим биссектрисы до пересечения с прямой / в точках В' и С риСг 48 (рис.48). Треугольники BABf и С АС равнобедренные, а точки Р и Q - середины отрезков ВВ' и СС . Поэтому прямая PQ параллельна ВС, отрезок QR - средняя линия в треугольнике CCBf, PR - средняя линия в треугольнике ВВ'С . Отсюда Ъ + с PR = ?_. — Ъ + с-а 106. 60 и 95. Пусть а и Ъ - искомые двузначные числа, а разность между суммой всех двузначных чисел и а + Ъ в 50 раз больше, чем а, т.е. 4905 - (а + Ъ) = 50я . Возможны три случая (а + Ъ < 200): а + Ъ = 55; а + Ъ = 105; а + Ъ = 155. В первых двух случаях одно из чисел а или Ъ не будет двузначным. 107. а) Может; б) п - нечетно. Из условия следует, что одна из девочек знакома со всеми мальчиками, одна - с (п - 1) мальчиком, ..., одна - с одним мальчиком и одна не знакома ни с одним из мальчиков. Общее число знакомств девочек равно 0 + 1 + 2 + .. . + п = — -. Если каждый из мальчиков знаком с k девочками, то общее количество знакомств мальчиков равно kn. Так как количество знакомств мальчиков равно п(п + \) количеству знакомств девочек, то ———- = kn , откуда k = —— . Так как k - целое число, то п нечетно. Докажем, что при любом нечетном п нужная система знакомств существует. Пусть в компании п = 2k - 1 мальчиков, 2k девочек и ни один мальчик не знаком ни с одной девочкой. Разобьем всех девочек (их - четное число) на пары и занумеруем эти пары числами от 1 до k. Первую девочку из первой пары знакомим со всеми мальчиками, а вторую не знакомим ни с одним из мальчиков. Первую девочку из второй пары знакомим с 2k - 2 мальчиками, вторую - с одним оставшимся мальчиком. Вообще, первую девочку из пары с номером / знакомим с п - I + 1 мальчиками, а другую - с / - 1 оставшимися мальчиками. В 66
результате все девочки окажутся знакомыми с разным количеством мальчиков, а каждый из мальчиков знаком с k девочками. 108. 5, + 52. Пусть ZBAE = а , АЕ = EF = FA = а (рис.49). Тогда ^a, ZCFE ^a. 6 3 Выразим площади 5t, 52 и иско- мую площадь S через а и а: 51 = 1 Рис. 49 1 2 2 - —a sin а cos а = — a sin 2 4 = j a2 sin (J- 2а 2а, Осталось заметить, что sin 2а + sin — 2а = sin 2а 109. -1 или 1. Перемножив равенства у-Z 2-Х У-X Х- у = £-— , у- 2 = Х-2 = получим уг хг ху )()2f. X у 2 Так как х Ф у , хф 2 , у * г , то х2у222 - 1. Итак, либо хуг = = 1, либо хуг = -1. НО. 3<я<12. Поскольку всякий угол многоугольника, удовлетворяющего условию задачи, равен либо 30°, либо 90°, либо 120°, а сумма всех внешних углов равна 360°, число п о/?ло вершин многоугольника не больше чем ——- = 12 . На рисунке 50 показаны примеры многоугольников с числом сторон от 5 до 12, удовлетворяющих условию. 111. 19962 -5. Пусть а = 1996. Тогда а - 3)(а - \)(а + \){а + 3) +16 = у/а4 -\0а2 +25 = а2 - 5. 67
АЛ п = 7 п = 9 я =10 Рис. 50 =11 n = 12 112. ad. Пусть М - середина стороны CD трапеции ABCD (рис.51). Площадь трапеции равна площади параллелограмма ABKL. 113. 11 очков. Если команда наберет 11 очков, она заведомо займет одно из первых четырех мест. Де- рис л сять набранных очков еще не обеспечивают выход в финал, как показывает пример турнира, в котором первые 5 команд играют все игры между собой вничью, выигрывают у последних трех команд и в итоге набирают по 10 очков. 114. См. рисунок 52. 115. а) 9876524130; 6) 1024375869. Пусть т - сумма цифр, стоящих на первом, третьем, пятом, девятом местах десятичной записи числа, а п - сумма остальных цифр. По признаку 68
а) б) Рис. 52 делимости на 11 число т - п делится на 11, т.е. т- п = \\k, где k - целое число. Кроме того, т + п = 45. Без труда доказывается, что в случае а) т = 28, п = 17, а в случае б) т = 17, п = 28. 116. 2т. На продолжении AM за точку М возьмем точку D так, что MD = AM (рис.53). Четырехугольник ABDC - параллелограмм с диагональю AD, равной А'А", поскольку треугольники А А'А" и ACD равны. 117. По крайней мере один из участников слукавил. Просуммируем отдельно числа, названные девочками, и числа, названные мальчиками. Очевидно, полученные числа должны быть равны. Однако это невозможно, так как одно из них делится на 3, а другое - нет. 118. 3333-3334 . Заметим, что Рис. 53 11112222 = 11110000 + 21111 = 111110002 = 3333 • 3334 Вообще при любом п: 1111... 1 • = 33...3 • 333...34. п-\ 119. а) Может. Для этого ему нужно делать совершенно произвольные ходы до последнего - 20-го хода. 20-й же ход он сможет сделать так, чтобы остаток от деления на 9 полученного 19-значного числа не дополнялся вторым игроком до 9. б) Нет. Если первый игрок на каком-то ходу пишет цифру а, второй приписывает цифру Ъ так, что а + Ъ = 6. 120. Чисел с суммой цифр 30 больше. Заметим, что если удовлетворяющее условию число оканчивается нулем, то наибольшая возможная сумма цифр не превосходит 9 + 8 + 7 + 6 = = 30. Поэтому десятичная запись чисел а, сумма цифр которых равна 31, не оканчивается нулем. Но тогда сумма цифр числа а - 1 равна 30. Итак, каждому из чисел с суммой цифр 31 мы однозначно поставим в соответствие число с суммой цифр 30. Но, например, числу 98643 не соответствует никакое число с суммой цифр 31. 121. При каких п число р - 2п +1 будет простым? 122. Тригонометрия. 69
123. См. рисунок 54. 124. Таких примеров неоглядно много. Побеждали в этом соревновании те, кто проявил наибольшую эрудицию. ■l:: 1+3+.. .+п •—•— •—•— • • • • 2+4 + 6+... Рис. 54 пуп + 1) 2 п\ + 2п = п(п+ 125. 17. Пусть целые а и b - стороны прямоугольника. Из условия следует, что 2а + 2b = ab, или a{b-2) = т.е. а = 2 + Ь-2' Есть 3 возможности: 6 = 3, тогда а = 6; b = 4, тогда а = 4; b = б, тогда я = 3. Итак, это прямоугольники 3x6 = 18, 4x4 = 16. 126. Существует. Например, число 235 • З35 • 584 • 790 удовлетворяет условию. Из условия следует, что п делится на 2, 3, 5 и 7. Будем искать п в виде п = 2k ■ 3k ■ 5l • 7т . При этом k должно делиться на 5 и 7, а /е + 1 - на 6, / - на 6 и 7, а /+ 1 - на 5. Наконец, т - на 5 и 6, а т + 1 - на 7. После сделанных замечаний нужные значения k, /, m подбираются без труда. 127. 30°, 60°, 90°. По свойству биссектрисы ~~ DM 1 , — = - (рис.55). Сле- Рис. 55 BD ВС 70
довательно, ZDCB = 30°. Пусть ЕК 1 CD. Тогда МК = СК. Это следует из того, что ЕМ = MB, a ZMEC = 2ZMEK = 60° . 128. (0; 0; 0); (0; 1; 1); (1; 0; 1); (-1; 0; -1); (-1; -1; 0); Перемножим первое и третье уравнения и возведем в квадрат второе: Приравнивая левые части, приходим к уравнению после преобразований имеем хУ2(х-у)2=0. Осталось рассмотреть случае х = 0, у = 0их = у. 129. 1842. Докажите прямым вычислением, что ап+5 - ап при любом п. Так как аХ9 = а4 = 19, а97 = а2 = 97 , мы можем вычислить я1998 = а3 . 130. Можно (рис.56). 131. Бесконечно. Поскольку уравнение можно переписать как (X-yf+(y-2f+(X-2f=6, его решениями будут например, х = п + 1, у = п, z = п - 1, где neN . г- п 2 Рис. 56 132. а) 1; 6) 2; в) \"\. Воспользуйтесь тем, что функция f = хх + ... + хп - ххх2 -х2х3 -... ... - хп-\хп ~ хпх\ ~ линейная по каждому из переменных хх, х2,..., хп . Зафиксируем xv..., xk_x, xk+x,..., хп и положим xk = х . Тогда /^ (л:) = kx + 1 достигает своего наибольшего значения либо при х = 0, либо при х = 1. Повторяя проведенное рассуждение, убеждаемся в том, что максимальное значение функции /"достигается, когда некоторые из Xj равны 0, а остальные 1. Остается выбрать наибольшее из значений f при таких х. 71
15 133. — 4 134. Составное, так как = (25 + 56)2 - (23 • 53)2 = (25 - 23 • 53 + 56)(25 + 23 • 53 + 56). 135. 7 136. yjP2 + Q2 . Пусть ABC - данный треугольник, CD - его высота, Pt - периметр треугольника ACD, Р2 - периметр треугольника BCD (рис.57). Поскольку каждый из треугольников ACD и BCD подобен треугольнику ABC, получаем пропор- р=7- Т = ~с- Откуда Рис. 57 а + о Р" с 137. Можно. Это следует из того, что = 1. =4. 138. 2500; 324; 100. Сторона k сотого квадрата - целое число (докажите это!). Если п - сторона исходного квадрата, то п2 - k2 = 99 . Осталось решить уравнение в натуральных числах. 139. — Существует угол ф, для которого а = coscp, Ъ = sin ф . Тогда аъЪ - b3a = abla2 - b2) = -sin2ф Максимум достигается при sin 2ф = 1. 140. Можно. Раскрасим клетки таблицы в 4 цвета (в клетках на рисунке 58 расставлены номера цветов). В каждом прямоугольнике 1x4 окажется по одной клетке каждого цвета. Далее перед всеми двойками и тройками поставим знак -. Сумма чисел в каждом прямоугольнике 1x4 равна 0, а общая сумма равна -6. Рис. 58 141. 1,125. 72 1 2 3 4 1 2 2 3 4 1 2 3 3 4 1 2 3 4 4 1 2 3 4 1 1 2 3 4 1 2 2 3 4 1 2 3
142. Можно. Разобьем все гири на пары с одинаковыми весами: (1; 30), (2; 29)... (15; 16). Пусть выбраны какие-то 10 гирь, с общей массой, равной — массы всех гирь. Поскольку построенных нами пар 15, из них можно выбрать 5 пар, не содержащих выбранных гирь. Они образуют первую группу. Вторую группу образуют гири, входящие в оставшиеся пары и не принадлежащие к сделанной выборке - их 10 штук, а 1 суммарная масса равна — общей массы всех гирек. о 143. Л.В.Канторович. 144. Л.Эйлер. 145. Трисекция угла, удвоение куба, квадратура круга. Все эти задачи были решены лишь в XIX веке. 146. В конце XIX века была создана теория множеств. Кроме того, аксиоматический метод стал основным в построении математических теорий. 147. Например, уравнение Пелля - им впервые занимался Ферма. Формула Кардано была открыта Тартальей. 148. 322357176. 149. См. рисунок 59. 150. 3; 5; 7. Если Л, Ь и с - искомые простые числа, то abc = l(a + b + с). Одно из чисел а, Ъ, с заведомо равно 7. Пусть <2 = 7. Тогда be = Ь + с + 7, или (6 — 1)(с — 1) = 8 , т.е. либо с - 1 = 2, Ъ - 1 = 4, либо Ъ - 1 = 4, с - 1 = 2. 151. —S. Заметим, что SABK =—S. Пусть Р - середина отрезка LC (рис. 60). Тогда МР - средняя линия треугольника BLC, a KL - средняя линия треугольника AMP. Поэтому PC = —1 ч ч 1— н Рис. 59 Рис. 60 73
= LP=AL,T.e. AL = ±BC, SABL = ±S и 152. (±372; — 3 ± 3V2j. Сложив уравнения системы, получим, что (х - у - З)2 = 0 , откуда х - у = 3. 153. Нельзя. Поскольку нужно перевезти 50 камней за 7 рейсов, по крайней мере на одном рейсе придется перевозить не меньше 8 камней (принцип Дирихле!). Однако, вычислив массу восьми самых легких камней, обнаружим, что ^^ она больше 3 т. D 154. 110°. Пусть Р- рис ^ точка пересечения прямых АВ и DE (рис.61). Отрезок BE - средняя линия треугольника APD. Поэтому АВ = = ВР = ВСи треугольник АРС - прямоугольный ( ZACP = 90° ). Точки F и С лежат на окружности с диаметром АР. Поэтому ZAPC = - ZB = 20° , ZAFC = 180° - 20° = 160° , а ZDFC = 360° - (90° +160°) = 110°. 155. В > А. Пусть а = 1998. Мы должны сравнить числа А = (а - If аа+{ (а + if"1 и В = {а - if"1 аа {а + if+1. Но это то же самое, что сравнить (а-\)а и (я + 1)2. P + Q 156. —~— . Площадь каемки из прямоугольников, окружающей заштрихованный прямоугольник, равна Q - Р. Половина этой площади содержится в параллелограмме KLMN. Поэтому +p 157. Нет. Это число делится на 5. 158. Равнобедренный и прямоугольный с катетами а = Ь. Пусть у "" угол между сторонами а и Ь. Тогда по известной формуле S = - ah sin у . Поскольку - ah sin у < —- < —-— , 2 2 2 4 равенство возможно лишь при sin у = 1, а = Ъ. 74
159. 12600. Разобьем все числа от 0 до 999 на пары: (0, 999), (1,998), ..., (499,500). Суммы цифр каждой пары равны 27. Осталось из числа 500 • 27 вычесть сумму цифр всех чисел от 0 до 99, которая равна 50 • 18 - 160. Нельзя. Сумма углов 14 треугольников меньше суммы углов 17-угольника. 161. Не играли. Пусть в турнире п участников. Играется —Ц-—- партий. В нашем же случае п - 2 участника полностью сыграли свой «подтурнир», а двое выбывших участников в первых пяти турах сыграли либо 9, либо 10 партий. Если они сыграли 10 партий, т.е. не играли между собой, то откуда п = 10. Если играли, получается уравнение (*-2>(*-3>+9 = 38, не имеющее целых корней. 3 Го3У 162. Нет. Пусть 6п = р6 , а 8п = q8 , тогда _ = £- , т.е. з 4 W) >/3 = 2^j - рациональное число. Противоречие! q 163. Л < В. Заметим, что А-1-- + --- --—-1-—<1 2 + 2 3+"' + 9 10" 10< ' D 1 11 11 1111, В = — + ... + — + — + ... + — + — + ... + — >- + — + —>1. И 20 21 30 31 40 2 3 4 164. 28 15 = 420. Число прямоугольников равно произведению количества пар вертикальных прямых на количество пар горизонтальных прямых. 165. От древнегреческих слов (дошедших до нас через латынь), означавших столик, еловая шишка, валик. 166. а2 = be . 167. Например: к, е, 0. 168. Трисекция угла, квадратура круга, удвоение куба, построение циркулем и линейкой правильного 7-угольника. 169. Например: Монж, Лаплас, Кондорсе, Карно, Улугбек, Вышнеградский, Валлис. 75
170. а) 2802; 6) 45я-10""1+1. Научимся сначала считать сумму цифр множества всех целых чисел от 0 до 99„. 9 . Для я цифр этого выпишем их все подряд по порядку и рассмотрим пары чисел, равноотстоящих от начала и конца записи: (0; 99...9) - первая пара, (1; 99...98) - вторая, ... я цифр я-1 цифра ..., (4 99...9 ; 5 00...0 ) - последняя пара. Сумма цифр чисел я-1 цифр я-1 цифра каждой пары в любом разряде одна и та же - она равна 9, разрядов всего п, так что сумма цифр чисел одной пары равна 9я, а количество пар равно половине количества рассматриваемых чисел, т.е. 5- 10й"1. Таким образом, сумма цифр всех целых чисел от 0 до 99...9 1 я цифр равна 45п 10 . Теперь легко решить обе части задачи. а) Подсчитаем искомую сумму цифр так: сначала всех чисел от 0 до 999 - она получается из доказанной формулы при п = 3 и равна 13500; затем - всех чисел от 1000 до 1999: для этого достаточно к 13500 (только что подсчитанной сумме цифр трехзначных «хвостов» этих чисел) прибавить 1000 - сумму единичек, стоящих в разряде тысяч); осталось прибавить сумму цифр числа 2000 - двойку. б) Искомая сумма отличается от уже подсчитанной в начале решения только на число 10" . Но сумма цифр этого числа равна 1. 171. (l + V2; 1/V2), (l - V2; - 1/V2). Сложив первое неравенство с удвоенным вторым, получим: [х2 - 4ху + 4у2) - 2 (х - 2у) + 1 < 0 , т.е. (х-2у)2-2(х-2у) + \<0. Итак, ((.г - 2у) -1) < 0 , что возможно лишь когда х - 2у - 1 = = 0. Почленная сумма двух нестрогих неравенств одного смысла дает равенство, только если оба исходных неравенства обращаются в равенства, поэтому мы приходим к системе уравнений х = 2у +1, 2у2 + 2у - х = 0 76
(мы обратили в равенство лишь второе из исходных неравенств). Решения этой системы и дают ответ. 172. 180°-а. Поскольку центр вписанной окружности лежит в точке пересечения биссектрис внутренних углов треугольника, а центр вневписанной - в точке пересечения биссектрис внутреннего и двух внешних его углов, ZIBJ = ZJCI = 90° (каждый из этих углов равен сумме половин двух смежных углов, а смежные углы дают в сумме 180°). Из этих равенств следует, что точки В, /, С и / лежат на окружности с диаметром // (рис.62). Для этой окружности искомый угол ВМС - центральный, а угол BJC - вписанный, опирающийся на ту же А* Рис. 62 дугу BIC, поэтому искомый угол вдвое больше угла В]С. Осталось найти этот последний угол. Заметим, что четырехугольник BJCI вписан в окружность, поэтому сумма его противоположных углов / и / равна 180°. Но т 180° 4-а А угол / равен как угол между биссектрисами двух углов треугольника, если третий его угол равен а . Осталось вычислить искомый угол. 173. а) 10; б) 11; в) —г— . Решим задачу в общем виде - для любого натурального п. Пусть к такое, что среди п человек нет двух поздравивших друг друга. Тогда общее количество поздравлений равно nk, п(п-\) , ^ п -1 ,_ _ . причем nk < — , т.е. k < . (Мы учли, что если бы каждый из имеющихся п людей поздравил каждого из остальных, т.е. (п - 1) человека, то всего поздравлений было бы 77
п{п-\) это соответствует тому, что мы взяли максимально возможное k, оно равно я - 1.) Если к > . п(п-\) , то nk > — , и значит, найдется пара поздравивших друг друга. Задача в) решена. Ответы в а) и б) получаются из в) соответственно при я = 20 и и = 21. Замечание. Заметим, что если k < , то можно так организовать поздравление, что никакие двое не поздравят друг друга. На рисунке 63 показано, как это может быть при п = 5, 1 2 3 4 5 1 - - + + 2 + + - - 3 + - - + 4 - + + - 5 + + k = = 2. В первом столбце табли- Рис. 63 цы идут номера поздравляющих, а в первой строке - кого они поздравляли. Знак «+» в клетке (а] Ь), где а - номер строки, где расположена эта клетка, а Ъ - номер ее столбца, показывает, что человек а поздравил человека 6, а знак «- » - что а не поздравил Ъ. Мы считаем, естественно, что сам себе человек обычно поздравлений не пишет, поэтому клетки (1; 1), (2; 2), ..., (5; 5) таблицы заштрихованы. Этот пример подсказывает, как можно поступить для любых допустимых я и k. Рассмотрим квадратную таблицы (я +1) х (я +1), где первая строка и первый столбец означают номера поздравляемых и поздравляющих соответственно, а «главная диагональ», идущая от клетки (1; 1) к (я; п) , заштрихована. Будем расставлять в таблице плюсы и минусы «антисимметрично»: если в клетке (а; Ь) стоит знак «+», то в клетке (Ь; а) поставим знак «-» (это и отражает «антисимметричность» ситуации - отсутствие взаимных поздравлений). В каждой строке должно быть k знаков «+». Условие k < гарантирует нам выполнимость желаемого заполнения таблицы: для каждого «плюса» найдется еще незаполненная симметричная относительно заштрихованной главной диагонали клетка для «минуса». 78
174. а) Можно; 6) можно. На рисунке 64, а показано, как разрезать на равнобедренные трапеции правильный треугольник, на рис.64, б и 64,в соответственно - как разрезать квадрат и равнобедренный прямоугольный треугольник на трапеции и правильные треугольники. а) Рис. 64 175. 4а. Докажем сначала равенство треугольников ABD и CMD (рис.65, а). По условию, BD = МС, AD = DM; кроме того, легко усмотреть равенство углов 3 и 4 - каждый из них смежный с одним из равных углов 1 и 2. Из полученного равенства следуют два важных следствия: А В = DC (1) - это очевидно; AN = ND = а (2) - это следует из равенства углов 6 и 7: равны углы 5 и 6, с одной стороны (соответствующие углы в равных по доказанному треугольниках), и углы 5 и 7, с другой стороны (они вертикальные). а) В б) Рис. 65 Итак, осталось найти отрезок DC, равный стороне АВ. Для этого выразим сторону через данный отрезок AN. Проведем прямую МР параллельно прямой CN (рис.65,б). Дальше все просто: отрезок МР - средняя линия в треугольнике BCN, а отрезок ND - средняя линия в треугольнике АРМ, откуда следует равенство отрезков AN, NP и РВ. Итак, А В = За. С помощью равенств (1) и (2) отсюда получаем ответ. 79
176. а) Можно; б) можно; в) нельзя. Проще всего начать со случая в). Заметим, что сумма I3 +23 +... + 20013 нечетна - в ней нечетное число нечетных слагаемых 13,33,53,...,20013 . Но при перемене знака у любого ее слагаемого k3 вся сумма уменьшится на четное число 2k3, поэтому ее нечетность сохранится. Если же менять знак у четного слагаемого рассматриваемой суммы, нечетность ее также сохраняется. Итак, из нечетной суммы I3 +... + 20013 нельзя переменами знаков получить четное число 0. Теперь рассмотрим параллельно случаи а) и б). Возьмем 16 последовательных кубов чисел k3\k +1)3,... ...,(& +15)3 и докажем, что можно так расставить знаки «+» и «-» между ними, что получится 0. Для этого рассмотрим 8 равенств (k фиксировано): k3 = k3, (k + 7)3 = k3 + 3k2 • 7 + 3k • 72 + 73. Заметим, что при любом / /2-(/ + 1)2-(/ + 2)2+(/ + 3)2=4, / - (/ +1) - (/ + 2) + (/ + 3) = 0 . Поэтому k3 -{k + \f ~{k + 2)3 +(k + 3)3 -{k + if +{k + 5)3 +{k + 6)3 - -{k + lf = 13_23+33-43+53+63-73=48 при любом k. Но это значит, что расстановка знаков перед кубами любых 16 последовательных целых чисел дает нулевую сумму (48 - 48 = 0). В случае а) разбиваем числа 13,23,...,19993 на группы по 16 последовательных кубов начиная с нуля (добавив слагаемое О3 ), а в случае 6) - начиная с начала - с 1. 80
177. 5. Если обозначить сумму чисел, вычеркнутых Борей, через х, получится, что сумма чисел, вычеркнутых обоими, равна Ах - делится на 4. Поскольку сумма всех данных 9 чисел равна 45, могло остаться либо число 1, либо число 5, либо число 9 (больших чисел среди девяти данных нет). Если осталось число 1, сумма оставшихся чисел равна 44, тогда Боря вычеркнул 4 числа, дающих в сумме 11, что невозможно (даже самые маленькие из оставшихся чисел (2; 3; 4; 5) дают в сумме больше). Если осталось число 5, Боря должен вычеркнуть в сумме 10, что возможно: такую сумму дают числа 1, 2, 3, 4. Тогда числа 6, 7, 8, 9 должен вычеркнуть Витя. Если осталось число 9, Боря должен вычеркнуть в сумме 9, что невозможно. S 178. ~. Из условия следует, что хорда МР параллельна диаметру АС полукруга (рис.66). Поэтому площади треугольников AMP и ОМР равны (у них общее основание МР и одинако- вые высоты, опущенные на МР). Значит, искомая площадь равна площади сектора ОМР, т.е. трети площади полукруга (по условию А" ^ радиусы ОМ и ОР делят полукруг на три равные части). Рис' 66 179. 27. Пусть рассыпанное число равно а6 . Поскольку сумма цифр а , равная 45, делится на 9, то и само число а делится на 9. Значит, число а делится на 3. Так как а6 - девятизначное число, а 26 = 64 , получаем: 206 = 64000000 < а6. С другой стороны, 326 = 230 = (210)3 = = 10243 > 10003 = 109 > а6. Итак, 206 < а6 < 326 , но а6 Ф 306 , так как в числе 306 шесть нулей, а у нас всего один нуль. Остались три кандидата: а = 21; а = 24; а = 27. Но число 217 оканчивается на 1, а такой цифры у нас нет. Число 246, как легко понять, возводя последовательно число 4 в натуральную степень, оканчивается на 6, что в нашем случае невозможно. Остается лишь число 27. Замечание. Поскольку в условии сказано, что рассыпана шестая степень числа, нет необходимости проверять цифровой состав числа 276. 81
;ос В 180. Да. См., например, рисунок 67. 181. а) Нет. Периметр данного многоугольника, очевидно, нечетен. Покажем, что для описанного многоугольника с четным числом С сторон такого быть не должно. pUCt (rf Пусть А^А2...А2п -описанный многоугольник, хьух - отрезки, на которые делит точка касания сторону АХА2; х2, у2 ~ сторону Л2Л3,...,; х2п1у2п -сторону А2пАх (рис. 68, а). По известной 901 Рис. 68 теореме о касательных, проведенных из одной точки к окружности, имеем: *1 = У2п > У2п-\ = Х2п . Сложив почленно все равенства, получим слева сумму сторон А{А2,А3АА,...,А2п_хА2п (обозначим ее через 5), а справа - сумму остальных сторон многоугольника: A2A3fAAA5f...A2nA^f она также оказалась равной 5. Таким образом, периметр многоугольника равен числу 25, а если длины всех сторон - целые числа, то периметр такого многоугольника - четное число. 82
Получено противоречие. б) Нет. Предположим, что такой многоугольник существует. На рисунке 68,6 изображен его фрагмент Ах999^2000\А2. Мы видим противоречие: с одной стороны, 2000 = 1999 + 1, а с другой - 2000 = х + у = ВА2000 + САХ < ^1999^2000 + \А2 - 2000 . 182. а) Можно. Например, последовательность {#„}, где k ak - , a fe = 1, 2, ..., 2000, - арифметическая прогрессия 2000! (напоминаем, что п\ = \ 2 3•... - (п - \) • п ). Действительно, п + \ п 1 2000! 2000! 2000! = const. 6) Нет. В арифметической прогрессии ап+х - ап = d - постоянная величина. Но в бесконечной последовательности вида — раз- 1 1 * ность соседних членов стремится к нулю. 183. г. Пусть ВВХ и CQ - высоты остроугольного треугольника ABC (рис.69, а). Тогда из треугольника АССХ получим, Рис. 69 что АСХ = 6 cos А , а из треугольника АСХН - что АСХ _ АСХ _ bcosA АН = cos ZBAH cos (90° - ZB) sin В Осталось заметить, что из теоремы синусов b = 2R sin В, где R - радиус круга, описанного около треугольника ABC. Тогда АН = = 2R cos A . Если угол А тупой, то буквы А и Н как бы меняются ролями 83
(рис.69,б): АВ{ = bcos(\SO° - A) = -b cos А (из Д/ЩС ); лтт ^Д ^#i bcosA , ААТТП УА у~ АН = 1 = 1 = - (из ААНВ< , где Zl = Z2 sin Z/ltfBj sin Б sin В - каждый из них в сумме с углом НСВ составляет 90°). Осталось заметить, что ——- = 2R (теорема синусов), sin В Если тупой угол В или угол С, все аналогично. Итак, AH = 2R\cosA\. Из условия задачи R = 2R\cosA\, откуда |cosA| = - и получается ответ. 184. .r + # + z = 3. Перепишем условие в виде системы: \х3-3х2=у3-3у\ (1) \уэ-3у2=г2-322. (2) Из уравнения (1) получаем: откуда {х- Поскольку х Ф у (по условию), имеем: х2 +ху + у2 =Ъ(х + у). (3) Аналогично получаем у2 +yz + z2 =3(y +г). (4) Почленно вычитая (4) из (3), получим: х2 + ху - yz - z2 = 3 (х - z), т.е. Сократив обе части полученного уравнения на х - z Ф 0, получаем ответ. Замечание. Задачу можно было решить проще, если воспользоваться теоремой Виета для кубического уравнения ах3 + Ъх2 + сх + d - 0 , имеющего 3 корня: xvx2,x3 . Согласно 84
этой теореме, Ъ хх+х2+х3 =—, а а ххх2 едзд а Если обозначить равное по условию значение х3 - Зх2, у3 - Ъу2 и z3 - 3z2 через Ъ, видим, что х, у и z - различные корни кубического уравнения t3-3t2-b = 0, поэтому первое из уравнений теоремы Виета сразу дает ответ. Доказать теорему Виета можно, например, так. Как известно, если х0 - корень многочлена fn (*) = %*" + "\Хп~Х + • - • + <*п-\х + ап , то fn (х) = (х-х0)- дп_х (х), где дп_х (х) - многочлен степени п-\. Поэтому для кубического многочлена с различными корнями хьхъхъ верно равенство ах3 + Ьх2 + сх + d = а (х - хх)(х - х2)(х - х3). Раскрыв скобки и приведя подобные члены в правой части полученного равенства, приходим к равенству ах3 +bx2 +cx + d = = ах3 - а [хх + х2 + х3) х2 + а (ххх2 + ххх3 + х2х3)х - аххх2х3. Приравнивая коэффициенты при одинаковых степенях х в левой и правой частях равенства, получим теорему Виета. Аналогично доказывается теорема Виета для корней любого многочлена я-й степени с п действительными корнями. 185. S. Рассмотрим равнобедренный прямоугольный треугольник ABC с высотой CD, равной /^ . Она делит треугольник на 2 равных треугольника, ACD и BCD, причем треугольник ЛВС подобен им с коэффициентом V2 (рис.70,а). Поэтому можно считать, что данный в условии сектор получен из треугольника ABC, а полуокружности построены на его катетах и пересекаются в точке D (рис.70,б). Очевидно, что сегменты, имеющие на рисунке 70, б номера 85
а) А D б) А В В Рис. 70 от 1 до 4, равны и имеют одинаковые площади — . Поэтому сумма площадей секторов 2 и 3 равна S. Заметим также, что S можно выразить как разность площадей полукруга CDB и треугольника CD В: 2 2 J { 2 \2 Теперь найдем искомую заштрихованную площадь: от площади четверти круга радиуса ВС отнимем площади треугольника ABC и секторов 2 и 3: = (тс - 2) - S = 25 - S = S. 186. 9 и 1. Поскольку а2 +Ь2 = 1, можно считать, что числа а и Ъ - координаты точки М окружности х2 + у2 = 1 радиуса 1 с центром в начале координат. Аналогично и и v - координаты точки N окружности х2 + у2 = 4 радиуса 2 с центром в начале координат (рис.71). Тогда данное в условии выражение - квадрат расстояния MN: f(xx)2+{yy)2 Рис. 71 {MNf=(xM-xN)2+{yM-yN)2 86
Из рисунка 71 видно, что MN наибольшее и наименьшее, когда М и N лежат на диаметре JV большей окружности: наименьшее, NMX, когда они в одной четверти, а наибольшее, NM2, - когда в разных. 187. Существует. На рисунке 72 центрами клеток квадрата 4x4 изображены члены компании из 16 человек. Шестеро друзей каждого из них расположены в той pUCt 72 же вертикали и той же горизонтали, что и он сам. Ясно, что каждые двое имеют в точности двух общих друзей. 188. Нет. Пусть • • • • • • А • • • А - аха2...ап = ■ 10я"1 + а2 ■ 10"~2 +... + ап_х ■ 10 + а„ (1) и одновременно А = ага2...ап . (2) Если в правой части записи (1) положить а2 = а3 - ... = ап = 0 , она уменьшится и получится, что А > ах • 10я"1. (3) Если в правой части (2) положить а2 - а3 - ... = ап - 9 , она от этого не уменьшится и получится, что А<аг9п~*. (4) Но (3) и (4) дают, что ах • 10я"1 < ах -9я"1, что, очевидно, неверно. Полученное противоречие приводит к ответу. 189. а2 - Ь2 = (а - Ь) (а + b). Вырежем из квадрата ABCD со стороной а (рис.73) прямоугольник ECFG со сторонами Ъ и а - Ь и переместим его в положение AHKL. Площадь прямоугольника HKFD, равная (a + b)(a-b), равна разности площадей квадратов ABCD и LBEG, равной а2 -Ь2. 190. а) Леонардо Пизанский по прозвищу Фибоначчи (сын Бо- наччи). Числа fn называются числами Фибоначчи. б) Если п делится на 4. В Ь Е К ъь L G F н Рис. 73 D 87
2 А о Последовательность fn можно задать так: ах = а2 = 1; ап+2 =ап+ ап+х при п > 1. Рассмотрев последовательность остатков от деления fn на 3, т.е. 1, 1, 2, 0, 2, 2, 1, 0, 1, 1, ..., нетрудно заметить периодичность этой последовательности с периодом 8. А так как fA = 3 и ^8=21 делятся на 3, то fn делится на 3 тогда и только тогда, когда п делится на 4. В 191. 2л. Переложите заштрихованные части рисунка 74, как > показано стрелками. В силу сим- к метрии получится, что искомая Рис. 74 площадь равна площади прямоугольника ОАВС. 192. — Восстановим рассуждения «некоторых математиков». Пусть искомая сумма равна S. Поскольку 1 -1 + 1 _ 14-1-... = 1 -(1-1 + 1-1 + ...), получим, что S = 1 - 5, откуда S = —. «Найдем» вторую сумму. Обозначим ее через X • Поскольку 1 - 2 + 22 - 23 +... = 1 - 2(1 - 2 + 22 - 23 +...), получим, что X = 1 - 2£, откуда X = —. о 193. Давид Гильберт. Речь идет о 3-й проблеме Гильберта: равносоставлены ли куб и правильный тетраэдр того же объема? Отрицательное решение получено в 1901 году немецким математиком Деном. 194. а) Да. Поскольку З2 +42 = 52, сумму 12032 +16042 = (3 • 401)2 + (4 • 401)2 = 4012 (з2 + 42) = = (5-401)2 =20052 можно заменить на 20052. б) Да. Аналогично п. а) можно заменить 12062 + 16082 = 52 • 4022 = 20102 на 20102. 195. а) 20°. Заметим, что угол В данного треугольника равен 60°, а треугольник AM С равнобедренный, причем ZAMC = 120° (рис.75). Поэтому если провести окружность радиуса МА = = МС с центром в точке М, то точка В окажется на этой 88
20 окружности. Но тогда MB - радиус этой окружности и из равнобедренного треугольника AM В находим, что ZABM = 20° . 6) 40°. Поскольку сумма углов В и М четырехугольника BDME равна 180°, около него можно описать окружность. В этой окружности впи- а санные углы МВЕ и MDE опира- рис у$ ются на одну и ту же дугу ME и поэтому равны. Но ZMBE = ZB - ZABM = 60° - 20° = 40° . 196. 3 или 7. Напомним, что если остаток от деления числа ах на Ь равен гх (т.е. ах = bqx + гх ), а для числа а2 он равен г2 (т.е. а2 = bq2+r2), где гх и г2 - целые числа от 0 (включая 0) до Ъ - 1 (включая Ъ - 1), то числа (Ц ±а2 и аха2 дают при делении на Ъ такие же остатки, как и числа гх±г2 и гх • г2 соответственно. Поэтому если остаток при делении числа р на число q равен г, чтобы найти остаток от деления, скажем, р2 -2 на q, достаточно найти остаток от деления на q числа г2 - 2 . Пользуясь этим, найдем остатки от деления чисел р - 2 , 2р2 - 1, Зр2 +4 на 7 в зависимости от остатка при делении на 7 простого числа р (рис.76). Поскольку все рассматриваемые числа простые, их делимость на 7 (остаток равен нулю) означает просто равенство семи. Мы видим, что во всех строках рисунка 76 имеется 0, что означает равенство нулю соответствующего числа. Если р = 1 (первая строка), остальные числа равны соответственно 47, 97 и 151 - простые. остаток от деления р на 7 0 1 2 3 4 5 6 остаток от деления р- 2 на! 5 6 2 0 0 2 6 остаток от деления 2р2- 1 на 7 6 1 0 3 3 0 1 остаток от деления Зр2+ 4 на 7 4 0 2 3 3 2 0 Рис. 76 89
Если Зр2 + 4 = 7 (вторая и седьмая строки), то р = 1 - не является простым. Если 2р2-\ = 7 (третья и шестая строки), то р = 2 - простое, р2 -2 = 2 - простое, Зр2 +4 = 16 - не является простым. Наконец, если р2 - 2 = 7, то р = 3 — простое и числа 2р2 -1 = 17 и Зр2 + 4 = 31 ~ тоже простые. 197. (0;0); {у[4]Щ. Решение (0; 0) очевидно. Пусть теперь у ф 0. Разделив обе части первого уравнения на у5, получим систему, равносильную данной при у Ф 0: х - У Пусть f (t) = t5 +1; g (t) - t3 +1. Обе функции - возрастающие (это легко установить - например, так: f = ЫА +1 > 0; д' = 3t2 +1 > 0 ), поэтому каждое свое значение принимают только один раз. Значит, последняя система равносильна такой: х _ У D ,2 = 2у L*< = 2у. Подставив х из первого уравнения во второе, находим, что у (г/3 - 2) = 0 , откуда (при у * 0 ) получаем, что у = у/2 . Приходим к ответу. 198. г. Пусть О - точка пересечения биссектрис углов А и В треугольника ABC (рис.77). Тогда ОС - диагональ квадрата со стороной г, образованного точками О, Си точками касания вписанной в треугольник ABC окружности. Поэтому ОС = rV2 . Далее, ZB = ZACD (каждый из этих углов в сумме с углом Рис. 77 90
27 А дает 90° - из треугольников ABC и ACD), поэтому Zl = Z5 и, значит, ZECB + ZEBC = 90° , следовательно, ZCEB = 90° . Аналогично ZAFC = 90° . Поскольку в четырехугольнике CEOF углы Е и F - прямые, около него можно описать окружность с диаметром ОС. Отметив, что ZECF = -ZC = 45° , с помощью теоремы синусов получим, что 15 EF = OC- sin 45° = rV2 • — = г . 199. а) 183. Ясно, что количество полосок не больше чем = 183 , а 183 полоски вырезать можно. Для этого из листа 27 х 34 следует вырезать угловой прямоугольник 7x9, оставшаяся часть без труда разрезается на полоски 1 х 5 , а из прямоугольника 7x9 вырезаются 12 полосок 1x5, причем остаются 3 клетки (рис.78). б) 152. Несмотря на то что 918 делится на 6, разрезать лист 27 х 34 на полоски 1 х 6 нельзя. Чтобы убедиться в этом, раскрасим клетчатую доску в 6 цветов, как показано на рисунке 79,а (цвета обозначены цифрами). Если бы доску можно было разрезать на полоски 1x6, клеток всех цветов было бы поровну. Однако вне углового прямоугольника 3x4 клеток всех цветов поровну, а в самом прямоугольнике - по одной клетке цветов / и 6, по две - цветов 2 и 5 и по 3 - цветов 3 и 4. На рисунке 79,6 показано, как из прямоугольника 9x10 можно вырезать 14 полосок 1x6. Далее (как и при решении пункта а)), можно вырезать из листа угловой прямоугольник 9x10 и оставшуюся часть (см. рис.79,в) разрезать очевидным образом на полоски 1x6. в) 131. Из листка 27x34 вырезается угловой квадрат 13x13 и разрезается так, как показано на рисунке 80. Поскольку 13x13 = 169 = 24-7 + 1, то больше 24 полосок выре- 7| J 25 34 Рис. 78 91
а) 30 3- 12 23 f ^ s 5 6 1 4 1 4 5 6 1 ft 1 12 Ямс. 79 Рис. 80 10 24 — - - - 1 13 зать из этого квадрата нельзя, а 24 мы смогли вырезать. 200. а) 4. Среди участников вечеринки имеется как человек А, сделавший 8 рукопожатий, так и В, не сделавший ни одного. Заметим, что миссис Браун не могла быть А - ведь среди остальных людей имеется В, с которым она должна была совершить рукопожатие как Л, но В ни с кем не пожимал руку. Пусть А - это мистер (миссис) Смит. Тогда его жена была человеком В (иначе В был бы среди других и А не смог бы пожать ему руку). Итак, А и В - пара Смит. Если удалить эту пару, для оставшихся 4 пар ситуация повторится, но число рукопожатий каждого уменьиштся на 1. Снова удалим «экст- 92
ремальную» пару супругов, и т.д. В конце останется пара Браун, причем окажется, что миссис Браун сделала 4 рукопожатия. 6) k - 1. Решение по индукции полностью аналогично п. а). 6 1 1 201. — часа. За час лев съедает 1 овцу, лев - —, волк - ~ . 11 Z о . 1 1 И Поэтому за 1 час они съедят 1 + —+ — = — овцы. 2 о Ь 202. Нет. По «основной теореме арифметики» любое натуральное число можно разложить, причем единственным образом (с точностью до порядка), в произведение степеней простых чисел: где числа Р\, ]%, - - •, рп ~ простые. Поэтому число а может иметь делителем лишь числа q = р1^ • р™2 •... • р™п , где одновременно выполнены все неравенства 0 < fWj < ^ ; 0 < т2 < h± ; ...; 0 < тп < kn , а тИ),^,...,тп ~ натуральные числа. Легко видеть, что количество делителей числа а равно произведению (fcj +1) (k2 +1) •... • (kn + 1) - ведь каждый сомножитель pf может входить в делитель в любой целой степени от 0 до 1% - как раз (k{ +1) вариант. Пусть а - данное трехзначное число. Тогда Поскольку 210 = 1024 - уже четырехзначное число, а число а трехзначное, получаем, что + An<10, (1) l)-...(kn + l) = 25. (2) Теперь заметим, что число 25 можно разложить на множители лишь двумя способами: 25 = 1 • 25 = 5 • 5 , поэтому из равенства (2) следует, что либо 1*2 + 1 = 25, либо ^+1 = ^+1 = 5. В первом случае имеем ^ = 24 и не выполняется условие (1), а 93
во втором получаем k\ = k4 = 4 . Тогда * = Р\ • г4 • Но если рх = р2 = 2, то а = 28 и имеет всего 9 делителей, а следующий вариант, когда рх = 2 , /?2 = 3 , дает д = 24 • З4 = 16 • 81 - четырехзначное число. В остальных случаях число получается еще больше. 203. с + hc > а + Ъ . Заметим, что hc • с = аЪ (1) (справа и слева стоит удвоенная площадь данного треугольника). Ясно, что (с - а)(с - Ь) > 0 (гипотенуза больше любого катета). Раскрыв скобки слева, получим с2 - с (а + Ь) + аЪ > 0 . Заменив теперь аЪ с помощью равенства (1), получим: с2-с(а + Ь) + + chc > 0, т.е. с - (а + b) + hc > 0 , от- I/ \Л куда получаем, что с + hc > a + b . Г Л 204. Да. Пример приведен на ри- Рис. 81 сунке81. 205. 1; 2; ...; 99; 101. Действительно, используя формулу суммы членов арифметической прогрессии для 99 чисел 1; 2; ...; 99, получим: 1 + 2 + ... ... + 99 = ^^-99 = 4950. Взяв сотым числом 101, получим искомые 100 чисел. 206. а) Нет. Поскольку сумма всех чисел сначала равна 21, а при каждой операции меняется на 2 - четное число, она никогда не станет четной. Но если все б чисел будут равны, их сумма четна. б) Нет. Пусть k раз к некоторым тройкам чисел прибавляли по 1, а / раз - отнимали. Тогда, если после этого все числа оказались равными а, получается, что 21 + 3(k -1) = 6а , откуда k - I = 2а - 7, т.е. k - I нечетно, но тогда и k + / нечетно. Рассмотрим прямоугольник с вершинами в точках, где вначале стояли числа 2, 3, 5 и 6. Любая тройка идущих подряд чисел имеет с этим прямоугольником ровно 2 общие точки. Поэтому каждая операция меняет первоначальную сумму чисел в прямоугольнике - число 16 - на 2 или на (-2), т.е. в конце она равна 16 ± 2т , где т - нечетное число. Поэтому указанная 94
сумма четна, но не делится на 4. С другой стороны, она должна равняться 4я, т.е. делиться на 4. Полученное противоречие дает ответ. 207. Нет. Рассмотрим, например, последовательность, задаваемую формулой ап - 2п . Предположим, что для нее выполнено условие задачи. Возьмем самую длинную сторону - ее длина равна 2". Тогда сумма остальных сторон должна быть больше - ведь отрезок, соединяющий концы этой длинной стороны, имеет меньшую длину, чем любая ломаная, соединяющая эти же концы, в том числе и сумма длин остальных сторон. Таким образом, при нашем предположении должно выполняться неравенство 2" < 2*1 + 2^ +... + 2*"1, (1) где ^,&2,...,&ш ~ целые числа, меньшие п. Но даже если остальных сторон п - 1, 2-'-1) 2-1 ~1 1<г • что противоречит неравенству (1). 208. Сумма всегда равна 0. Представим исходную таблицу в виде суммы двух таблиц (рис.82). Если расставить знаки «-» в соответствии с указанным правилом, сумма чисел в каждой из складываемых таблиц будет равна 0: в каждой строке первой таблицы все 10 чисел одинаковы, поэтому если взять пять из них со знаком «-», сумма 10 чисел этой строки станет равна 0. Аналогично рассуждения для равных 10 чисел каждого столбца второй таблицы. 209. Получится 2001384. Если приписать искомые 3 цифры, получится семизначное число, которое делится на 7-911 = 693 . Разделим 2001000 на 693 «уголком» с остатком: 2001000 = 2887-693+ 309. Значит, осталось к числу 2001000 прибавить число 693 - 309 = = 384. Замечание. Можно решать задачу и с использованием признаков делимости. Решение получится не короче. 0 10 80 90 0 10 80 90 ... 0 ...10 ...80 ...90 1 1 1 2 .. 2 .. 2 .. . 10 . 10 . 10 Рис. 82 95
210. 7 команд. Пусть некоторая команда Л проиграла команде В (такая команда заведомо найдется - это следует из условия) - на рисунке 83,а этот факт показан стрелочкой, ведущей от В к Л. Тогда по условию найдется такая команда С, которая выиграла и у Л, и у В (рис.83,я). Но тогда для команд а) А< С D Рис. 83 Л и С по условию найдется такая команда D, которая выиграла у обеих этих команд (рис.83,б; команда В для этой роли, очевидно, не годится). Итак, команд, выигравших у Л, не меньше трех (и это для произвольной команды Л!) Но среди всех команд имеется такая (назовем ее X), которая выиграла не меньше партий, чем проиграла. (Иначе, если все команды выиграли меньше, чем проиграли, число проигрышей будет больше числа выигрышей, что в волейболе невозможно - там нет ничьих.) Эта команда X не меньше трех раз проиграла и, значит, не меньше трех партий выиграла, поэтому у нее было не менее 6 противников. Поэтому число участников было не меньше 7. Пример турнира из 7 команд показан на рисунке 83,в. Команды изображены вершинами семиугольника на плоскости. Стрелки направлены от команд-победительниц к побежденным. 211. #32 > #3 + #5. Пусть * = #3 + #5. (1) Возведем обе части этого равенства в куб, используя формулу (a + bf = a3+b3'+3ab(a + b), получающуюся из обычной группировкой утроенных произведений. Получим: х3 = 3 + 5 + (Мы заменили сумму в последнем слагаемом на левую часть исходного равенства (1).) 96
Итак, число х = ^3 + \/5 - корень уравнения Выясним знак левой части этого уравнения при х = у/32 . Для этого подставим это значение х в левую часть: (№)3 -ЗШ5 .$/32 -8 = = 24 - 3^/15 • Ш = 24 - 6^/60 = б(4 - З/бО) > 0 , так как 4 = Ш > . Итак, если f (х) = х3 - зЩЕх - 8 , то [Г{Щ>0. Заметим, что Г(х) = Зд:2 -3^/15 = З(х2 -Щ = З(х-Щ(х График f'(x) изображен на рисунке 84,д, из которого следует, что при х0 = у/\5 функция f(x) имеет минимум, а при х > х0 а) \ I б) Рис. 84 возрастает (см. рис.84,б). Осталось заметить, что f\yf32j > 0 и поэтому соответствующая точка графика f(x) находится правее корня ^/З + ^5 , что и дает ответ. Можно поступить и так. Сначала доказать, что при любых а > 0 и Ь > 0 справедливо неравенство a+Ъ ^ Ча+Ш из которого следует, что 2 4 Библиотека Квант 97
212. Все углы по 60°. Пусть а, Ъ и с - стороны треугольника ABC, та , ть и тс - его соответствующие медианы. Учтя, что в точке пересечения медиан они делятся в отношении 2:1, считая от вершины, из условия равенства периметров (см. рис.85,а) получим, что 2та 3 2mb 3 2' откуда -(та - = -{а-Ъ). Но в треугольнике большей стороне соответствует меньшая медиана (это следует, например, из известной формулы для Рис. 85 медианы: та = — V2 (b2 +с2)- а2 , так что полученное равенство возможно лишь если та =ть и а = Ъ (иначе левая и правая части имеют разные знаки). Итак, а = Ъ, и треугольник ЛВС - равнобедренный. Осталось использовать условие, что четырехугольник AMON вписан в окружность. Из равнобедренности треугольника ABC следует, что треугольники ONM и CNM равнобедренные и ОС - ось симметрии четырехугольника ONMC. Поэтому ОС - диаметр окружности, значит, медианы BN и AM (рис.85,б) - высоты равнобедренного треугольника ABC, и он оказывается правильным. Значит, каждый его угол равен 60°. Замечание. Утверждение, что ОС - диаметр круга, можно доказать и так. ОМ = ОМ => Zl = Z2 , СМ = СМ => Z3 = Z4 , поэтому ZCMO - ZCMO , а поскольку сумма этих углов равна 180°, каждый из них равен 90°. Поэтому ОС - диаметр. 98
213. Это бюст знаменитого математика и философа Рене Декарта, фактически высказавшего идею условного рефлекса. 214. Это число 17, знаменитое еще и тем, что правильный 17-угольник можно построить циркулем и линейкой. Действительно, пусть 2(х + у) = ху (слева стоит периметр прямоугольника со сторонами хиг/,а справа - его площадь). Из этого уравнения легко получить, что * х-2 х-2 х-2 (это преобразование называется выделением целой части дроби). Осталось заметить, что для того чтобы у было целым числом, надо, чтобы число х - 2 было делителем числа 4 (тогда второе слагаемое последней суммы будет целым числом). Таким образом, либо х - 2 = 1, либо х - 2 = 2, либо х - 2 = 4, т.е. либо х = 3, либо х = 4, либо х = 6. В первом случае г/ = 6, во втором у = 4, в третьем г/ = 3. Итак, либо хг/ = 18, либо ху = 16. 215. Предположим, что многочлен с целыми коэффициентами р (х) = аохп + аххп~х +... + ап - генератор простых чисел. Очевидно, что ап Ф О. Если \ап\ > 1 , то при всех достаточно больших целых к, делящихся на |я„| , число p(k) будет составным. Если \ап\ = 1 и р(1) - простое число, то рассмотрим многочлен p(k + 1) = kq(k) + p(l). При достаточно больших k, делящихся на р(1), р(& + 1) делится на р(1) и не является простым числом. 216. Гиппократ пытался решить задачу о квадратуре круга, т.е. о построении циркулем и линейкой квадрата, равновеликого данному кругу. Решена эта задача была в 1882 году Линде- маном, доказавшим трансцендентность числа п и тем самым установившим неразрешимость задачи о квадратуре круга. Задачи о луночках решаются так. Пусть катеты треугольника ABC равны а и Ь, а гипотенуза - с, половина площади описан- пс2 n(a2+b2) ного круга равна = — , сумма площадей луночек 8 8 равна na\nb2 (n(a2+b2) ) Q * q \ q О О I О 99
Аналогично решается и задача о трапеции: если АВ = ВС = = CD = я, то 4 — — Sabcd - Sabcd • 8 V l ) 217. Отрицательные числа. 218. a) 22003-l цифра; б) 9-22002. Умножим число A-ax...ak, где av...,ak*0 - цифры, на 99...9, n>k. Имеем я В = А (10" -1) = А ■ 10" - А = a^...ak00^0 - ax...ak п Выполняя вычитание «столбиком», получим ; n-k Это число (п + k) -значно, а сумма цифр его равна 9п. Поэтому количество цифр в десятичной записи произведения равно +... + 22002 =22003-1, а сумма цифр равна 9 • 22002. 219. h2. Повернем треугольник BCD вокруг вершины В на 90° так, чтобы точка С попала в точку А (рис.86,я). Так как в исходном четырехугольнике ZB + ZD = 180°, то и ZA + ZC - 180° . Поэтому точка D при повороте попадет в точку D' на продолжении AD за точку А. б) В D Рис. 86 Площадь ABCD равна площади равнобедренного прямоугольного треугольника DfBD > в котором известна высота /г, опущенная из вершины прямого угла В. Эта высота разбивает его на 2 равных равнобедренных прямоугольных треугольника с катетами h (рис.86,б). Поэтому его площадь равна 100
220. з|- . Пусть у = 2х3 + х - 3 , тогда х3 = У * + 3 . Тог- я ^ я я о У - * + 3 да из данного уравнения у = 3 - * , значит, г/ = 3 - , т.е. х = 2у3 + у - 3 . Осталось решить систему уравнений |г/ = 2х3 + л: - 3, \х = 2г/3 + у - 3. Вычитая из первого уравнения второе, после очевидных преобразований получаем уравнение {х-у)(х2+ху + у2 + \) = 0. Второй множитель заведомо не равен нулю поэтому х = у. Подставив х вместо у в первое уравнение системы, приходим к уравнению 2х3 = 3 . 221. а) Нет; 6) нет; в) нет. Пусть п-\, п, п+\- последовательные целые числа. Сумма их квадратов равна Зп + 2 и при делении на 9 имеет остатки 2 или 5 (это можно установить, перебрав, например, все возможные варианты: п = = 3fc; n = 3k + I; n = 3k + 2). В то же время куб или сумма кубов нескольких последовательных чисел может дать при делении на 9 лишь остатки 0, 1 или 8 - также устанавливается перебором. Полученное противоречие дает ответ. 222. 120°. Известно, что биссектриса каждого внутреннего угла треугольника пересекается в одной точке с биссектрисами двух внешних углов при двух других его вершинах (рис.87,я). Рис. 87 101
В нашей задаче точка L является точкой пересечения биссектрис внутреннего угла ЛВК и внешнего угла ЛКС треугольника ЛВК (рис.87,б). Поэтому ЛЬ - биссектриса внешнего угла при вершине Л этого треугольника. Если ZA = 2а (рис.87,б), то получаем, что За = 180°, откуда а = 60°, ZA = 120°. 223. За 11 рейсов. Если груз представляет собой 44 ящика с о 36 9 „ массой — т = — т, то меньше чем за 11 рейсов его перевезти 44 11 нельзя (больше четырех ящиков четырехтонка за 1 рейс не перевезет). Покажем, как можно перевезти весь груз за 11 рейсов. Сначала будем загружать ящики в машину по одному до тех пор, пока масса груза в кузове не превысит 4 т. Снимем после этого последний положенный ящик, отложим его в сторону и отправим машину. Затем повторим такую процедуру 7 раз. Останутся 8 отложенных ящиков и еще сколько-то ящиков с а) б) Рис. 88 общей массой, меньшей 4 т. Очевидно, что все оставшиеся ящики можно увезти за 3 рейса. 224. Пусть, для определенности, т < п • Разрезать прямоугольник т х п на уголки можно тогда и только тогда, когда т х п делится на 3, кроме случаев: т = 1, п любое; т = = 3, п нечетно. На рисунке 88,а показано, как разрезать на уголки прямоугольник 2x3. Если тхп делится на б прямоугольник можно разрезать на (т, п отличны от 1), прямоугольники 2x3 и, значит, на уголки. При нечетных т > 5 , п > 9 вырежем из прямоугольника тхп угловой прямоугольник и разрежем его на уголки, как показано на рисунке 88,6. Оставшаяся часть разрезается на части 2 х 3 и тоже режется на уголки. 225. Через б ч. Пусть скорость выехавшего из Л равна х км/ч, выехавшего из В у км/ч и время до встречи t ч. Тогда xt = 9у; yt = Ах, перемножив эти уравнения, получим после сокращения на ху, что t2 = 36 . 102
226. Да, может. Пример приведен на рисунке 89. Ясно, что на нем легко так подобрать положение точек А, В, С и D, чтобы сумма площадей треугольников А ВВХ была треугольника ^, DCDX и меньше площади ВСС 227. Нет. Поскольку 16016003 + 1 = 40022 , т.е. 16016003 = = 40022 - I2 = 4001 • 4003 , данное число составное. 228. 17. Мысленно проведем все диагонали я-угольника. Из каждой из п вершин выходят п - 3 диагонали (они не идут в 2 соседние вершины, куда идут стороны). Таким образом, из вершин выходят п(п - 3) отрезка. Но каждая диагональ сосчитана пока дважды - у нее 2 конца. Поэтому число диагоналей п(п-3) равно . В нашей задаче —Ц-—- = 119, откуда п2 -Зп -238 = 0 . Положительный корень этого уравнения дает ответ. 229. 5. Заметим, что в каждом месяце не менее 4 пятниц (поскольку в нем не менее 4 недель). Если бы в году было хотя бы 6 месяцев с 5 пятницами, то в нем было бы не менее 6x5 + 6x4 = 54 пятниц, но в нем лишь 52 недели и еще 1 или 2 дня, так что эта ситуация невозможна. Покажем, что 5 месяцев с 5 пятницами может быть. Так будет, например, в високосном году, если 1 января - пятница (проверьте!). 230. На прямой / возьмем точку О и проведем полуокружность радиусом ОА с центром в точке О (первая линия), пересекающую / в точках К и L. Затем раствором циркуля, равным КА, проведем окружность с центром в точке L, пересекающую полуокружность в точке А' (вторая линия). Наконец, линейкой проводим прямую АЛ' (третья линия). Докажем, что KL \\ АЛ'. Из построения следует, что треугольники АОК и A'OL равны (ОК = ОА = ОА' = OL ; АК = A'L), поэтому равны их высоты АН и А'Н', значит, НАА'Н'- параллелограмм (его противоположные стороны АН и А'Н' равны и параллельны). Следовательно, НН' || АЛ'. 103
231. а > Ь. Известно, что для положительных чисел х и у справедливо неравенство — > фсу . Поэтому после почленного извлечения квадратного корня из данного неравенства (это законно, поскольку обе его части, очевидно, положительны) получаем: 2 2 откуда 1 < а — Ъ +1, т.е. b < а. 232. Можно. Путь нужной тройки команд нет. Возьмем любые 2 команды А и В и будем считать, что А выиграла у В. Но тогда А выиграла и у всех команд, проигравших В, т.е. А не брала больше очков, чем В. Таким образом, любые две команды набрали по разному числу очков. Но тогда найдется команда, набравшая ровно 7 очков. Противоречие. 233. 22 года. Эта олимпиада проводилась в 2002 году. Так как 442 = 1936 < 2002 , то ближайшие большие, чем 2002, квадраты - это 452 =2025 и 4б2 =2116. Ясно, что племяннику меньше 114 лет, поэтому 2116 и все большие квадраты отпадают. Остается, что ему будет 45 лет в 2025 году, а в 2002 ему было 22 года. 234. 8 раз. При заданном движении каждый обгон означает, что обгоняющий прошел за время с предыдущего обгона ровно на 1 круг больше обгоняемого. Пусть vA, vB , и vc - скорости мотоциклистов, t - время, по прошествии которого они оказались в одной точке, / - длина дороги. Тогда vAt - vBt - 4/, vBt - vct = 5/. Сложив почленно эти равенства, получим: vAt - vct = 9/. Значит, А прошел на 9 кругов больше, чем С, и, следовательно, обгонял его 8 раз. 235. а); 6) Существует. Пусть а = 2k + 1 - нечетное натуральное число, а а2+Ь2=с2. Тогда с2 -Ь2 = = (с - Ь) (с + Ь) = (2k + 1) . Подберем Ь и с так, чтобы выполнялись равенства \с-Ь = \, Треугольник со сторонами я = 2fc + l, b = 2k2+2k, c = 2k2+2k - искомый. 104
В частности, для а = 2001 получаем k = 1000 ub = 2002000; с = 2002001. v 2 _ 1 1 7 2 5 1 1 11 ) +;6)=++ + + Возможны и другие представления. 1 111 Например, вместо — можно написать — + т^Г+ 77Г • Ш z(J оО ЬО 237. 6) sina + sinp = 2sin^±-&cos^£ ZMAB = a , ZMBA = р (рис.90). Тогда ZKMA = длим отрезок AM за точку М на „ расстояние MB' = MB и заметим, что ZBMB' = a + Р . Продлим КМ до пересечения с ВВ' в точке Р. Поскольку ZB'MP = , МР - биссект- Рис 90 риса в равнобедренном треугольнике ВМВ', а следовательно, и высота этого треугольника. Поэтому треугольник КВВ' - равнобедренный и KB' = = KB = К А , т.е. треугольник АКВ' тоже равнобедренный и АН = НМ + MB' = ИМ + MB . Пусть R - радиус окружности. Тогда AM = 2Rsin$ , ВМ = 2#sinct , АН = AKcosZKAM = 2Rsin^±-£cos Итак, sin a + sin Р = 2 sin !-cos -. 238. Воспользуемся тождеством an-bn ={a-b)(an-x + я""2Ь + яп'3Ь2 +... + abn~2 + б""1 справедливым для всех натуральных я (его можно доказать, например, воспользовавшись для второй скобки в правой части формулой суммы п первых членов геометрической прогрессии с первым членом an~x и знаменателем — ). Предположим, что 2" -1 - простое число, а п - не простое, т.е. составное число. 105
Пусть п = kp, где k > 1, р > 1. Тогда - составное число, вопреки условию. 239. Это великий норвежский математик Нильс Хенрик Абель (1802-1829). Речь идет о его теореме о неразрешимости в радикалах алгебраических уравнений степени п > 5. 9091 240. п . Преобразуем числитель: УУО1 1010111110101 = 1012+1010 4+100 + 1 = 9 • 1012 + 9 • 1010 + 109 -104 +900 + 9 109-9091-9091 = 9091111111111. Аналогично преобразуется знаменатель. 241. Jab . Обозначим углы, как показано на рисунке 91. Получим следующие соотношения: Zl + Z3 = а (а - внешний угол для треугольника ADC), Z2 + Z4 = а ; Z3 + Z4 = а . Из первого и третьего равенств получим, что Zl = Z4 , а из второго и третьего - что Z2 = Z3 • Тогда треугольники ADC и BCD подобны, значит, AC DC Рис. 91 DC ВС ' откуда получаем ответ. 242. (0; 0); (1; 1); (1; - 1); (-1; 1); (-1; - 1). Дважды воспользовавшись неравенством a + b> 2\[ab для положительных чисел а и 6, которое превращается в равенство лишь при а = Ь, получим Надо найти такие пары [х\ у), при которых оба неравенства в выписанной цепочке обращаются в равенства. 106
Первое из них обращается в равенство, если x>(l + x>) = y2(i+y2), (1) а второе - если Рассмотрим функцию f(t) = t \\ + t ]. Она четна, поэтому ее график симметричен относительно оси ординат. Кроме того, f'{t) = 2t(\ +12), откуда следует, что при t < О она убывает, а при t > О - возрастает. Поэтому f(x) = f(y) только в следующих трех случаях: х = у = 0; х = у; х = -у. Условия (2) дают \х\ = \у\ = 1. Отсюда следует ответ. 243. а) 1. Если имеется многочлен f(x) = аохп +а\Хп~* +... ... + ап_хх + ап , то, чтобы найти остаток от деления числа f(q) на число t, достаточно найти остаток от деления числа f(r) на число t, где г - остаток от деления на t числа q. Нам понадобится еще формула a2k+] + b2k+' =(a + b) (a2k - a2k~xb + a2k~2b2 -... - ab2fi~] +b2k). Доказать ее можно, например, использовав то, что вторая скобка справа - сумма 2k первых членов геометрической про- о* ( ъ грессии с первым членом 2 и знаменателем — Итак, пусть п = 2k + 1 - нечетное число, п > 2, и число 5" + 4" ~ квадрат. Тогда 5"+4"=52Л+1+42*+1 = = (5 + 4)(52* - 52*"1 • 4 + 52*"2 • 42 -... + 42*). Рассмотрим остаток от деления последнего произведения на 8. Первый множитель, 9, дает остаток 1. Во втором множителе все слагаемые, начиная с третьего, делятся на 8. Осталось найти остаток от деления на 8 числа 52к - 52*"1 • 4 = = 52*"1 (5 - 4) = 52*"1. Возводя последовательно 5 в нечетную степень: 5, 125; 3125, ..., получим, что остаток при делении на 8 все время равен 5 (иначе: чтобы получить очередную нечетную степень, мы все время умножаем предыдущую на 25, а это число при делении на 8 дает в остатке 1). В то же время, если возводить остатки от деления нечетных чисел (1, 3, 5, 7) в 107
квадрат, получатся числа, дающие в остатке при делении на 8 только единички. Получено противоречие. Итак, указанное в условии число не может быть квадратом при нечетном п. Пусть теперь п четно: п = 2k. Найдем остаток от деления числа 52k + A2k на 3. Так как 4 дает остаток 1, второе слагаемое дает вклад, равный 1. Первое слагаемое дает такой же остаток, что и 22k = Ak , т.е. тоже 1. Итак, 52k + A2k дает при делении на 3 остаток 2. Но квадрат числа может дать при делении на 3 лишь остатки 0 и 1: если п дает остатки 0, 1 или 2, то п имеет соответственно остатки 0, 1 и 1. Опять противоречие. Остался лишь случай п = 1. Он дает квадрат. 6) 1; 2. Аналогично п. а) проверяется, что если п>2 нечетно, то число 52*+1 - 42Л+1 дает при делении на 8 остаток 5 и потому не является квадратом. Пусть теперь п = 2k - четное число, k > 2 • Тогда из равенства 52k - A2k = m2, где т, очевидно, нечетно, следует (5kf -т2 = 2Ак , откуда (5* -т)(Ьк + т) = 24к , т.е. р + q = Ak. Вычитая из второго равенства первое, получим, что 2т = 2я - 2Р, но т нечетно, поэтому p=\,q = 4k — \. Тогда (сложив первые два равенства системы) получим 2-5* = 2 + 24*"1, откуда 4 • 5* = 4 + 24* . Но 4 • 5* < 16* = 24* < 4 + 24* при k > 2 . Полученное противоречие приводит к ответу, который непосредственно проверяется. 244. 45°, 135°. Докажем, что Н - точка пересечения высот (ортоцентр) треугольника ЛВС. Пусть АХ,ВЬСХ - центры данных окружностей (точки В и С лежат на окружности с центром Ах и т.п.) - см. рисунок 92,а. Поскольку АХВСХН - ромб, ВАХ \\ НСХ (1). Аналогично, из того, что BXAQH - ромб, следует, что ВХА || НСХ, (2). Из (1) и (2) вытекает, что АХВ \\ ВХА , а так как при этом АХВ = ВХА (это радиусы одинаковых окружностей), АВХАХВ - параллелограмм. Заметим теперь, что СН _L AXBX - общая хорда двух окружностей перпендикулярна линии их центров (известный факт). Но АХВХ || АВХ, значит, СН 1 АВ, т.е. СН - часть высоты треугольника ABC. 108
а) Рис. 92 Аналогично доказывается, что АН _1_ ВС . Далее (сравните с задачей 183), треугольника АСНХ (рис.92,б) АНХ = Л С cos A = bcosA; а из треугольника АННХ получаем, что АН = АН, АНХ АНХ _ Ь cos A cos ZHXAH cos (90° -В) sin В sin В ' Ъ а Поскольку (теорема синусов) ——— = -—j » получаем, что a cos A ^ . АН = = a ctg A . sin Л Если же /Л - тупой, то АН = - a ctgA, т.е. АН = a |ctg/4|. Но по условию, a \ctgA\ = а, т.е. |ctg^| = 1. Значит, ZA = 45° или ZA = 135°. 245. 16. Пусть п - искомое число, т - число депутатов, голосовавших «за». Тогда 68 т 69 100 < п < 100 ' «Перевернем» все дроби: Ш п_ 100 _ 25 ~69~ < т < "б^ ~ 17' Отнимем от всех трех частей по единице: 31 п-т 8 — < < — 69 т 17" Снова проделаем эти же операции, только на этот раз целая 109
часть дробей равна 2, поэтому поэтому 17 8 1 п Зт п - ее и отнимем: т 69 ^ -т 31 ' -2п 7 -т 31 ' значит, 31 п-т 8. 7 Зт-2п Поскольку Зт - 2п > 1, получаем, что п-т> 5 (иначе не выполняется левое из последних неравенств), откуда п > 16 (можно получить эту оценку, подставив т = п - 5 в 68 т 69 ч ттт < — <77^г)- Число 16 удовлетворяет условию (тогда 100 п 100 т= И). 246. а) Можно. Задача решается по индукции по т + п. Для т + я = 2 утверждение очевидно. Пусть для любых /иип таких, что m + n<k и чисел av...,am; bvb2,...,bn искомую таблицу можно построить. Если даны числа ava2,...,am и Ь1,62,...,Ь„+1 строим таблицу тхп для чисел а^,...,ат и bXib2f...ibn =bn +bn+1, после чего добавляем (п + 1)-й столбец так, чтобы выполнялось условие задачи. 6) Можно. Здесь тоже можно было бы воспользоваться индукцией. Мы же поступим иначе. Пусть даны два набора положительных чисел ava2,...,am и bv...,bn, для которых а\ + а2 + • • • + ат = Ь\ + ^2 + • • • + \ ~ S • Рассмотрим отрезок длины S, на котором отложим последовательно отрезки длиной а^а2,...,ат . Затем на том же отрезке отложим отрезки &!,Ь2,...,ЬИ . Пусть a{j - длина отрезка, являющегося пересечением i-ro отрезка из отрезков ах,...,ап и /-го из отрезков b^...}bm . В таблицу тхп следует поместить число а^ в пересечение г-й строки и /-го столбца. Очевидно, суммы чисел в строках и столбцах полученной таблицы удовлетворяют условию. 3 247. В И ч 48 мин. На рисунке 93 показано, что за — часа группа прошла 3 одинаковых участка пути, а забывший компас пробежал 5 таких же участков - один назад и 4 вперед. 3 1 3 Поэтому он тратил на 1 участок • = ч = 18 мин . Зна- 110
чит, он прибежал назад в И Ч 30 МИН + 18 МИН = Ич НчЗОмин 12ч Л2ч 30м1ЙГ 13ч = 11 ч 48 мин. Рис- 93 248. Можно. Покажем, что любой треугольник можно разрезать на равнобедренные. Известно, что в него можно вписать окружность. Поскольку касательные, проведенные из вершин до точек касания, равны, 3 треугольника, образованные вершинами исходного треугольника и точками касания, равнобедренные (на рисунке 94 они заштрихованы). Осталось провести радиусы в точки ка- Рис. 94 сания. Так как любой выпуклый я-угольник можно разрезать на треугольники (например, проведя из какой-нибудь вершины все возможные диагонали - их (я - 3) штуки), ответ на вопрос задачи утвердительный. 249. (20032 + 2003 +l) . Пусть а = 2003. Подкоренное выражение имеет вид а2 + а2 (а + I)2 + а2 + 2а + 1 = [а2 + а + l)2. 250. Нет. Пусть в трапеции нижнее основание а, верхнее - Ь, боковые стороны cud, с > d . Отрежем от нее параллелограмм со сторонами bud (рис. 95). Поскольку сторона треугольника больше разности двух других его сторон, получим, что а - b > с - d. Поэтому условие задачи невыпол- нимо. а а-Ъ 251. 1; 2; 3; 4; 6. Для всех дан- Рис- 95 ных в ответе чисел утверждение задачи очевидно. Покажем, что при натуральных п > 7 условие не выполняется. Если п = 2k + + 1, k > 3 , его можно представить так: очевидно, 2 и 2k - 1 - взаимно простые числа: нечетное число 2k - 1 не делится на 2. Пусть n = 2k- четное число. Тогда п = Am или п = Am + 2. В первом случае числа 2т - 1 и 2т + 1 взаимно просты, а во втором - числа 2т - 1 и 2т + 3. ill
252. Можно. Числа 1; 3; 5; ...; 2003 образуют арифметическую прогрессию, одно из основных свойств которой - равенство сумм пар ее членов, равноотстоящих от концов: 1 + 2003 = 3 + + 2001 = ... = 1001 + 1003. Сложив по 2 полоски указанных длин, получим прямоугольники 1 х 2004 ; осталось приложить их сторонами 2004. Получится прямоугольник 1002x2004. 253. Нам надо построить угол —, имея угол — . Заметим, 71 271 П что — -— = — . Отсюда построение: 1) строим угол — (например, строим правильный треугольник); 2) удваиваем данный угол (это построение известно); 3) отнимаем от — угол —. о 7 254. В 1,5 раза. Пусть скорости отца и сына равны соответственно v0 и vc, периметр катка d, время, через которое отец обогнал сына при движении в одном направлении, равно t. При таком движении за время t отец проезжал ровно на d больше сына, а при встречном движении они за время — вместе проезжали d, т.е. Разделив первое уравнение на второе, получим 5- откуда _ 3 255. Прав. Пусть имеется треугольный торт, разрезанный биссектрисами на б кусков, причем один из этих кусков - прямоугольный треугольник. Прямой угол не может быть при вершине исходного треугольника - тогда угол этого треугольника был бы равен 180°, что невозможно. Если прямой угол при точке пересечения биссектрисы со стороной - например, угол 1 на рисунке 96,я, то и угол 2 - прямой, т.е. Холмс в этом случае прав. 112
а) В Рис. 96 Предположим, что прямым является угол (р на рисунке 96,6. Тогда из треугольника ЛОВ имеем: где Л, В, С- углы треугольника ЛВС. Получается, что Z.C = 0° , что невозможно. 256. Сумма четвертых степеней не меньше суммы кубов. Пусть х > 0. Сравним разность х4 - х3 с разностью х2 - х : х*-х3-(х2-х) = х2(х2-х)-(х2-х) = = [х2 - х)[х2 - l) = х(х + 1)(х -1)2 > 0. Осталось записать неравенства х2{ - xf > х2 - х{ для каждого i = 1, 2, ..., п и почленно сложить их. 257. Об А.Н.Колмогорове см., например, статьи в журнале «Квант», №3 за 2002 г., в газете «Математика», №23, 38, 39 за 2002 г. 258. Траектория движения каждой точки подвижной окружности - диаметр большой окружности. Пусть точка К малой окружности переехала из положения Кх в положение К2 (рис.97); при этом Ох и О2 - положения центра малой окружности, О - центр большой, Л - точка касания во второй момент. Мы используем далее лишь то, что вписанный в окружность угол измеряется половиной дуги, на которую опирается, а центральный - целой дугой. Так как радиус подвижной окружности в 2 раза меньше, /.К-^Л = 2/.К£)Л - ведь дуги КХА и К2Л равны - проскальзы- Рис. 97 113
вания нет. С другой стороны, точка О лежит на подвижной окружности, поэтому ZK2OA = - ZK2O2A = ZKfiA , значит, точки К2, Кх и О лежат на одной прямой. 259. Для доказательства достаточно проверить равенство (а2 + Ъ2)(с2 + d2) = (ас + bdf + (ad - be)2. 260. Окружность радиуса 2 с центром на продолжении отрезка АВ за точку В на расстояние 1. Выберем систему координат так, чтобы точка А была ее началом, а точка В имела координаты (3; 0) - см. рисунок ->• 98. Зная формулу расстояния АВ на координатной плоскости АВ = 1{ ~ *в? + (У а ~ У в)2 > за- ис' пишем условие задачи для произвольной точки М(х\ у), удовлетворяющей условию: 2 2 * +У = 2. Возведя обе части уравнения в квадрат, после приведения его к общему знаменателю и приведения подобных членов получим д:2-8* +г/2+ 12 = 0, откуда Получен ответ. 261. Пусть у многогранника в каждой вершине сходятся по крайней мере 4 ребра, а каждая грань имеет не меньше 4 сторон. Занумеруем грани многогранника. Пусть щ,п2,...,пг - количество сторон у этих граней. Тогда щ + п2 + ... + пг = 2Р (каждое ребро считалось дважды - по одному разу в каждой из двух граней, которым оно принадлежит) и 4Г < 2Р , т.е. 2Г < Р. Пусть т],т2,...,тв - количество ребер, исходящих из вершин многогранника. Поскольку тх + т2 + ... + тв = 2Р (опять каждое ребро считается дважды - для каждого из двух его концов), аналогично предыдущему имеем 2В < Р. Складывая полученные неравенства, приходим к противоречию с теоремой Эйлера: В+Г<Р. 114
262. а) Может. Например, 1111111112 =12345678987654321. 6) Нет. Если число оканчивается на 789, то при делении на 8 оно дает в остатке 5, а квадраты нечетных чисел дают остаток 1 при делении на 8 (см., например, задачу 243). 263. 2R sin 2а- Поскольку треугольник ВОС равнобедренный, ZOBA = ZOCB = - - а . Поэтому ZB = ZC = 2а и ЛВ = 2R sin 2a по теореме синусов. 264. (l; 0; >/2 j. Применим к каждому слагаемому левой части уравнения известное неравенство для любых чисел а и Ь: 22 ао < , причем равенство возможно лишь при а = о: Сложив почленно эти неравенства, получим: ху\\ - у2 + р\г - z2 + z\l3 - х2 < 3 , причем равенство возможно, лишь если Из этих равенств, в частности, следует неотрицательность х, Почленно возведем все уравнения в квадрат: У2=2~г\ z2=3-x2. Подставим z2 из третьего уравнения во второе: у2 = х2 -1; тогда из первого уравнения х2 = 1 - (х2 -1), откуда %2 = 1 и, 115
учитывая, что х > О , имеем х = 1. Тогда г2 = 2 , у2 = Q , и получается ответ. 265. а) 3. Начнем с важного соображения. Пусть т - некоторое натуральное число. Выпишем последовательно остатки от деления чисел Фибоначчи на т. Предположим, что k - наименьший номер, при котором число fk делится на т. Тогда последовательность остатков имеет вид 1, 1, 2, ..., я, О, а, а, ... и разбивается на группы по k чисел, заканчивающиеся нулем. Так, вторая группа из k чисел получена умножением остатков 1, 1, 2, ..., а на а. Отсюда следует, что числа fk , делящиеся на т, образуют периодическую последовательность. Т.е. все числа fkl и только они делятся на т. Начинаем выписывать подряд числа Фибоначчи: 1; 1; 2; 3; 5; ... Первое число Фибоначчи, делящееся на 9, это /j2 = 144 , причем на 9 делятся все числа вида fX2n » но они же (и только они) делятся на 16. 6) 2 и 8. На 16 делятся числа вида fx2n , но они же делятся на 9. 266. Пусть х - длина трети проведенного отрезка. Введем и разъясним обозначения, показанные на рисунке 99,а. По теореме о касательной и секущей, проведенных из одной точки к окружности, СК2 =CPCN = 2x2; но ML2 = MN • МР = 2х2, поэтому С К = ML . Равенство BL = В К очевидно. Итак, ВС = = МВ. Рис. 99 116
а) Нет. Если СМ - высота, то невозможно равенство ВС = = MB (гипотенуза ВС больше катета того же треугольника ВМС). б) Может. Воспользуемся формулой, выражающей медиану тс к стороне с треугольника через его стороны а, Ь, с: Am2 + с2 = 2 (я2 + Ь2) - см. рисунок 99,6: 4СМ2 + АВ2 =(АС2+ВС2), т.е. откуда, заменив 2х2 на у2, получим, что у = 4г. Таким образом, получается, что треугольник ABC имеет стороны АВ = = 102, АС = 13г, ВС = 5г. Без ограничения общности можно положить z = 1, тогда АВ = 10, АС = 13, ВС = 5, СМ = 6^2 . Осталось убедиться в том, что медиана СМ делится вписанной окружностью на 3 части длины 2>/2 . Пусть вписанная окружность пересекает медиану СМ в точках N и Р, причем CN = a, NP = b; тогда РМ = б>/2 - а - Ъ (рис.99,в). Легко найти расстояния от вершин треугольника до точек касания: как известно, расстояние AL, например, равно р - ВС, где р - полупериметр треугольника ABC; аналогично ищутся и другие отрезки (см. рис.99,в). Осталось записать известный факт, что если из одной точки проведены к окружности касательная и секущая, то квадрат касательной равен произведению секущей на ее внешнюю часть. В нашей задаче это 2 уравнения: CK2=CPCN и MI? = MN • МР , из которых все получается. в) Не может. Если СМ - биссектриса, то ZCMB = ZACM (см. рис.99,а), что невозможно, так как в этом случае АС \\ АВ . '■ -[§]• 267. - — , где [а] - целая часть а - наибольшее целое число, не превосходящее а. Легко показать, что (xx+x2+... + xnf = = хх + х2 +... + #„ +2\ххх2 + хххъ +... + хп_ххп). Или, в сокращенных обозначениях, i=l ) Ы\ l=i<; 117
Поэтому 1=1 При четном n = 2k минимум достигается, например, при x2i-i = 1» •Г2г = 0 . При нечетном - при хх = 0 , лг2 = 1 , лг3 = 0 , ..., д:„ = 0 . 268. а) Нет; 6) нет; в) да; г) да. Предположим, что лист бумаги удалось разрезать на фигурки нужного вида. Покрасим черным цветом клетки листа, как показано на рисунке 100.а. а) 6) 1 Рис. 100 Каждая из фигурок содержит не более одной покрашенной клетки. Пусть х - число «уголков», а у - число «зигзагов». Тогда х + у > п2 , Зх 4- Ay = (2n - I)2, откуда х > An - 1. Отсюда следует, что требуемых разрезаний не существует при п = 2 и /2 = 3. Например, при п = 3 потребовалось бы не меньше 11 «уголков». Однако в квадрате 5x5 всего 25 клеток. Если п = = 4, то .г>15. Значит, для квадрата 7x7 понадобится в точности 15 уголков и 1 «зигзаг». Такое разрезание возможно (рис. 100,6). Как «продолжить» разрезания на большие п, показано на том же рисунке. 269. 5 ч. Пусть х км/ч и у км/ч - скорости пешехода и велосипедиста соответственно. Тогда через час после начала движения пешеход и велосипедист находились в точках С и D соответственно (рис.101), а через - часа встретились, поэтому — (х + у) = х, откуда у = Зх. О6- С D у 4 , tg щее время движения пешехода рав- ЛВ 2х + у 5х ^ , ч х Рис. 101 118
270. Нет. Пусть а и Ь - натуральные числа, а + b = 30030. Тогда ab = a- (30030 - а). Заметим, что 30030 = 30 • 1001 = = 2-3-5-71113. Чтобы произведение аЪ делилось на 30030, надо, чтобы оно делилось на каждый из делителей этого числа. Ясно, что число а не может содержать их все. Пусть а = ар...у, где а,р,...,у ~ какие-то из делителей числа 30030. Тогда где 5,...,ф - остальные делители числа 30030. Очевидно, вторая скобка не делится на эти остальные делители, так что аЪ не делится на 30030. 271. 11. Разрежем доску на 16 квадратиков 2 х 2 . В каждом из них должны быть заняты по меньшей мере 2 клетки (иначе из квадратика можно вырезать «уголок»). Поэтому должны быть заняты 32 клетки, т.е. число «уголков» не меньше 11. А 11 уголков вырезать можно (рис.102). 1 L I L I L I L I L г 1 _ 1 J 1 J 1 J 1 J 1 J Рис. 102 л_л I Зтс Зтс л i (Зтс Зтс 5тс 1 (Зтс Зтс 2тс i ,_, . . 272. —;—;— ; —;—;— ; —;—;— . Пусть аа] {8 8 4/ {и 7 U) [\0 10 5 ) У 1 и AA2 - трисектрисы угла А, а BBX и ВВ2 - угла В (рис.103). Возможны 3 разных случая (с точностью до обозначений). 1) Высоты - отрезки ААХ и ВВ{ (как на рис.103). Тогда из треугольника АВВХ получаем, что ЗР + ос = , а из треугольника ВААХ - что За + Р = - . Из этой в, Рис. 103 системы уравнений находим, что а = р = -.Тогда ZA = ZB = — 8 8 2) Высоты - отрезки АА2 и ВВХ. Аналогично первому случаю, из треугольников АВА2 и АВВХ получаем систему = — ; ZC = n-(ZA 8 ZB) = - . 4 2' 119
откуда а = —, 14 Так получен второй ответ. 3) Высоты - отрезки АА2 и ВВ2. Аналогично получаем третий ответ. хг\Ю0 /£\100 273. 7100 больше. Достаточно сравнить I ~ I +1 - 1 с единицей. Но <г\100 /с\100 /г\3 /с\3 4ос 5^ +f?l <fil +f?l =^+ 7) [7J {7) {7) 343 343 343' 274. Можно. Пример: 2, 2, 2, 2, -9, 2, 2, 2, 2, -9, 2, 2, 2, 2, -9, 2, 2. 275. Делится. Пусть х = 250. Тогда 4*4 + 1 = 4д:4 + 4д:2 + 1 - 4х2 = (2х2 +2х + \){2х2 - 2х +1). Осталось заметить, что Ах4 + 1 = 2202 + 1, а первый сомножитель - это как раз 2101 + 251 +1. 276. а) Первый игрок. Можно считать, что точки делят окружность на равные части. Тогда точки ах и я1002 лежат на одном диаметре, и первый игрок может их соединить. После этого первый игрок применяет «симметричную» стратегию: на каждый ход второго отвечает проведением хорды, симметричной только что проведенной хорде второго игрока относительно диаметра из первого хода. 6) Первый игрок. Первым ходом первый соединяет точки с номерами 1 и 3. Тем самым он «выключает» точку с номером 2 - из нее теперь нельзя провести хорду - и сводит задачу к 2002 оставшимся точкам. Расположим эти 2002 точки для наглядности в вершинах правильного 2002-угольника (номера диаметрально противоположных точек будут при этом иметь разную четность). На каждый ход второго игрока первый отвечает ходом, симметричным относительно центра окружности. ка 277. . Обозначим АОХ через R, СО2 ~ через г. Тогда 4 АВ = 2(R + г) - диаметр большей (внешней) полуокружности, R + г - ее радиус. Найдем площадь арбелоса. Для этого надо от площади внешнего полукруга радиуса R + г отнять площади внутренних 120
O2 В полукругов радиусов R и г: л(# + г)2 kR2 кг2 ^-^- — -— = nRr. Заметим теперь, что треугольник ADB (рис.104) - прямоугольный (угол ADB - вписанный в окружность и опирается на ее диаметр АВ), a CD - его высота (CD - общая касательная для внутренних кругов, она перпендикулярна радиусам О2С и ОХС, проведенным в точку касания С). По известному свойству высоты прямоугольного треугольника, 2 CD2 = АС ■ СВ , т.е. а2 = ARr , откуда Rr = — . Подставляя 4 это соотношение в значение площади арбелоса, получим ответ. 278. Гаусс пытался выяснить, действительно ли сумма углов треугольника равна 180°, т.е. применима ли в реальной практике геометрия Евклида. 279. sin(cc + P) = sin ос cos P +sin P cos a . 280. Достаточно просто перебрать все возможные случаи остатков от деления х и у на 6. Например, ели х дает остаток 2, а у - остаток 3, то *2 дает остаток 4, у2 - остаток 3, а х2 л-хул-у2 - такой же остаток, как 22+2-3 + 32=13 - т.е. остаток 1. Аналогично во всех других возможных случаях получится остаток 1 (возможных - которые могут быть, если р - простое число; например, случай четных х и у исключается). 281. Речь идет о Пьере Симоне Лапласе, которого Наполеон Бонапарт назначил министром внутренних дел, а потом не знал, что с ним делать. 282. Нет, ибо В п+\ п-\ 283. 90°. Проведем окружность с центром в точке В и радиусом АВ (рис.105). Так как Рис. 121
и у] у - 1 > — , имеем ZAMC = 30°, точка М лежит на этой окружности. Поэтому ВС = СМ и ZBCM = ZBMC = 75°, a ZCBM = 30° . 284. х = у = 2. Ясно, что х > 1 , у > 1. Поскольку >/# - 1 ^ — - 1 + г/л/л* - 1 > ху , причем равенство возможно лишь при yjy - Т = — и у/х -\ = — . 285. а) Можно; 6) нельзя. Так как 1 + 2 + 3 +...+ 30 = 465, то на б групп с одинаковыми суммами числа разбить нельзя. На 5 групп разбить можно. Для этого каждую из трех групп 1,2, ..., 10; 11, 12, ..., 20; 21, 22, ..., 30 разбиваем на 5 пар с одинаковыми суммами, а затем полученные пары объединяем в 5 групп по б чисел в каждой. /2 286. — . Пусть прямая AL пе- h ресекает описанную окружность треугольника ABC в точке N (рис. 106). Центр О этой окружности лежит на прямой MN, а четы- рехугольник NMAH - паралле- Рис. 106 лограмм ( NM \\ АН и NM = = ЛЯ, так как ML = LH по условию). Точка L ~ середина AN. Поэтому OL J_ AN • В прямоугольном треугольнике OLN гипотенуза ON = R, катет NL = /, a NM = h - его проекция на ON. Поэтому /2 = Rh . 287. 2. Так как а = а3 -1, то За2 - 4а = За2 - За - а = За2 - За +1 - а3 = (1 - а)3. Аналогично, поскольку а3 = а + 1, то За2 + 4а + 2 = За2 + За + а +1 +1 = а3 + За2 + За +1 = (а +1)3. 288. Нет. Пусть на плоскости даны N точек, D - наибольшее, ad- наименьшее из расстояний между ними. Возьмем любую из данных точек и построим круг с центром в этой точке (рис.107). Все остальные точки окажутся внутри или на границе этого круга. Теперь рассмотрим круги радиусом с центрами во всех данных точках. Эти круги не перекрываются и их nNd2 n суммарная площадь равна . С другой стороны, все эти 122
круги целиком содержатся в круге радиусом D + — , поэтому их суммарная площадь меньше площади nNd2 большого круга, т.е. —-—< Следовательно, откуда D Рис. 107 -\ . Однако при N = 225, D < 21 и d > 3 полученное неравенство не выполняется. 289. Доля голубоглазых среди блондинов больше. Пусть Б - количество блондинов, Г - количество голубоглазых, М - общее количество людей, ГБ - количество глубоглазых блонди- ГБ Б ГБ Г нов. По условию — > — . Но тогда — > — . Г М Б М 290. Это основание высоты, опущенной на гипотенузу. Для всякой точки М на гипотенузе АВ прямоугольного треугольника ABC (рис.108) МК2 + ML2 = LK2 = МС2 . Искомый минимум достигается, когда минимальна длина МС, т.е. когда МС - высота треугольника ABC. 291. За4 +1 = (а2 - af + (а2 + of + (а2 - if . 292. 90°. Пусть F - точка пересечения прямых / и АВ. В треугольнике ABE отрезки ВС и EF - высоты, a D - ортоцентр (рис.109). Поэтому прямая AD перпендикулярна BE. П Рис. 108 Рис. 109 123
293. 5. Пусть т - возраст младшего. Тогда одно из чисел т, т + 2, т + б, т + 8, т + 12 и т + 14 делится на 5. В этом легко убедиться, рассматривая остатки от деления т на 5. А из условия простоты уже легко следует, что т = 5. 294. Да. Это вершины правильного пятиугольника и его центр. 295. V41 . Так как у1х2 - 6х + 13 + \1х2 - 14х + 58 = yj(x - З)2 + 2 - 7)2 0 fM(x,O) т данная функция равна сумме расстояний от точки М(х; 0) до точек Л(3; 2) и В(7; 3) соответственно (рис.110). Наименьшее значение этой суммы равно расстоянию от точки А' (3; -2) до точки В(7; 3). В самом деле, если А' (3; -2) - точка, симметричная точке А относительно оси Ох, то АМ + МВ = А'М + МВ, но последняя сумма минимальна, когда точки А', М и В лежат на одной прямой. 296. 3. Пусть а - первая цифра десятичных записей чисел 2п и 5" . Тогда при некоторых k и / а • 10* < 2п < (а + 1) • 10* , а • 10' < 5й < {а + 1) • 10'. Перемножив эти неравенства, получим А' Рис. 110 т.е. a2 <\0n-k~l <(a + \f. Поскольку а - цифра, это возможно лишь при а = 3, п - k - I = = 1. 297. 12. а + Ъ 298. . Пусть М и N - середины сторон АВ и DC соответственно ( рис .111). Четырехугольник KMEN - параллелограмм,
площадь которого равна -SABCD . Треугольники КАЕ и KNE, а также КМЕ и KDE равновелики. Поэтому АВ \\ ЕК \\ CD, т.е. ABCD - трапеция, а КЕ - ее средняя линия. 299. 50 мин. Человек встретил машину за 10 минут до прибытия поезда, на котором он обычно приезжал, ибо в обычной ситуации машине пришлось бы еще проехать от места встречи до вокзала и обратно, затратив на это 20 минут. Значит, от вокзала до места встречи с пешеходом машина едет 10 мин, а пешеход идет 50 мин. 300. 2 решения. Очевидно решение х = у = z = 0. Пусть х, у, г - ненулевое решение системы, тогда обязательно х < 0 , у < 0, z < 0. Пусть (для определенности) х < у < z < 0 . Тогда x2>y2>z2 и х4 > у4 > z4 . При этом х + у2 + z4 < z + х2 + у4 = 0 . Поэтому если не все числа х, у, z равны между собой, то х + у2 + z4 < 0 . Отсюда следует, что х = у = z < 0. Уравнение же х4 + х2 + х = 0 , т.е. (при д:*0) *3+* + 1 = 0, имеет единственный отрицательный корень. 301. Например, Блез Паскаль, Декарт, Ферма, Валлис, Грегори, Броункер. С некоторой натяжкой Ньютон и Лейбниц. 302. Нет, ибо х4 +1 = х4 + 2х2 + 1 - 2х2 = = (д:2 - W2 +1)(*2 + xyf2 303. V2+V3+V5. 304. Доказательство легко следует из рисунка 112. Рис. 112 125
305. — . Пусть о Тогда Отсюда 5 = — 8 З2 п2 . 1 1 1 с 1 тс 1 + + + + + 5 + .2 д ,2
СОДЕРЖАНИЕ Предисловие 3 I олимпиада 5 II олимпиада 8 III олимпиада 10 IV олимпиада 12 V олимпиада 15 VI олимпиада 18 VII олимпиада 21 VIII олимпиада 23 IX олимпиада 25 X олимпиада 28 XI олимпиада 31 XII олимпиада 34 XIII олимпиада 37 XIV олимпиада 40 Ответы, указания, решения 43
Андрей Александрович Егоров, Жозеф Михайлович Раббот Олимпиады «Интеллектуальный марафон». Математика Библиотечка «Квант». Выпуск 97 Приложение к журналу «Квант» №5/2006 Редактор А.Ю.Котпова Обложка А. Е. Пацхверия Макет и компьютерная верстка Е.В.Морозова Компьютерная группа Е.А.Митченко, Л.В.Калиничева 82 Формат 84x108 1/32. Бум. офсетная. Гарнитура кудряшёвская. Печать офсетная. Объем 4 печ.л. Тираж 3500 экз. Заказ 119296 Москва, Ленинский пр., 64-А, «Квант» Тел.: (495)930-56-48, e-mail: admin@kvant.info Отпечатано в ОАО Ордена Трудового Красного Знамени «Чеховский полиграфический комбинат» 142300 г.Чехов Московской области Тел./факс: (501)443-92-17,(272)6-25-36, e-mail: market ing@chpk.ru
индекс 70465 Библиотечка КВАНТ «Интеллектуальный марафон» МАТЕМАТИКА ВЫПУСК